Laboratory Testing Flashcards

1
Q

A 36-year-old woman presents to the emergency room with severe abdominal pain,
nausea, vomiting, anorexia, and somnolence.
ABG: pH 7.20, PCO2 35 mmHg, pO2 68 mmHg on room air
Laboratory values: Na 130 mEq/L, Cl 80 mEq/L, HCO3 10 mEq/L
1. How do you diagnose a simple acid–base disorder?

A
  1. Initially the pH is used to determine acidosis or alkalosis, and then the value of
    PaCO2/HCO3 is used to determine if the derangement is metabolic or respiratory.
    If it is of respiratory origin, then we will have to determine whether the process
    is acute or chronic. If it is due to a metabolic component, then respiratory com-
    pensation should be calculated using the appropriate formula.
How well did you know this?
1
Not at all
2
3
4
5
Perfectly
2
Q

A 36-year-old woman presents to the emergency room with severe abdominal pain,
nausea, vomiting, anorexia, and somnolence.
ABG: pH 7.20, PCO2 35 mmHg, pO2 68 mmHg on room air
Laboratory values: Na 130 mEq/L, Cl 80 mEq/L, HCO3 10 mEq/L
2. What blood gas abnormality does this patient have?

A
  1. Our patient has a pH less than 7.4, which signifies acidosis. The HCO3 is less
    than 24 mEq/L; therefore the primary abnormality in this patient is metabolic
    acidosis. This chart (Fig. 36.1) shows the steps to follow in order to diagnose an
    acid–base disorder [1].
How well did you know this?
1
Not at all
2
3
4
5
Perfectly
3
Q
  1. How do you calculate anion gap and corrected anion gap?
A
  1. Anion gap (AG) = Na − (Cl + HCO3)
    (a) AG is the difference in the ‘routinely measured’ cations (Na) and ‘routinely
    measured’ anions (Cl and HCO3) in the blood and depends on serum phos-
    phate and albumin concentrations [2]. Determination of AG is useful in deter-
    mining the cause of acidosis [3, 4]. The normal value for serum AG is usually
    8–12 mEq/L. In our patient, AG = 130 − (80 + 10) = 40 mEq/L. So, this
    patient has a high AG, most likely due to starvation or diabetic ketoacidosis.
    (b) In a normal healthy patient, negatively charged albumin is the single largest
    contributor to the AG [5]. Hypoalbuminemia causes a decrease in AG; hence
    AG is corrected to albumin level using the equation of Figge as follows: cor-
    rected AG = AG + [0.25 × (44 – Albumin)] [6].
    • If corrected AG >16, there is high AG acidosis.
    • If corrected AG <16, non-AG acidosis.
How well did you know this?
1
Not at all
2
3
4
5
Perfectly
4
Q

A 36-year-old woman presents to the emergency room with severe abdominal pain, nausea, vomiting, anorexia, and somnolence. ABG: pH 7.20, PCO2 35mmHg, pO2 68mmHg on room air Laboratory values: Na 130mEq/L, Cl 80mEq/L, HCO3 10mEq/L
4. How do you diagnose a mixed acid–base disorder and does this patient have
mixed acid–base disorder?

A
  1. Delta gap formula can be used to assess mixed acid–base disorder.
    (a) Δ gap = AG − 12 + HCO3 (12 is normal serum AG value)
    • If Δ gap <22 mEq/L, then concurrent non-gap metabolic acidosis exists.
    • If Δ gap >26 mEq/L, then concurrent metabolic alkalosis exists.
    (b) In our patient, Δ gap = 40 − 12 + 10 = 38 mEq/L. So, there is a concurrent metabolic alkalosis probably from vomiting in addition to high AG metabolic acidosis in this patient.
    So, there is a concurrent metabolic alkalosis probably from vomiting in addition to high AG metabolic acidosis in this patient.
How well did you know this?
1
Not at all
2
3
4
5
Perfectly
5
Q
  1. What is Winter’s formula?
A
  1. Winter’s formula is used to determine whether there is an appropriate respiratory
    compensation during metabolic acidosis [1].
    (a) Winter’s formula: PCO2 = (1.5 × HCO3) + 8
    • If measured PCO2 > calculated PCO2, then concurrent respiratory acido-
    sis is present.
    • If measured PCO2 < calculated PCO2, then concurrent respiratory alkalo-
    sis is present.
How well did you know this?
1
Not at all
2
3
4
5
Perfectly
6
Q
  1. Is there any compensation in this blood gas value?
    ABG: pH 7.20, PCO2 35 mmHg, pO2 68 mmHg on room air
    Laboratory values: Na 130 mEq/L, Cl 80 mEq/L, HCO3 10 mEq/L
A

Winter’s formula: PCO2 = (1.5 × HCO3) + 8
○ In our patient, calculated PCO2 = (1.5 × 10) + 8 = 23 mmHg according to Winter’s formula.
Our measured PCO2 of 35 mmHg is higher than the calculated PCO2 of
23 mmHg, so our patient also has concurrent respiratory acidosis. Usually, metabolic acidosis is compensated by respiratory alkalosis, but due to somno-lence in this patient, concurrent respiratory acidosis exists.

How well did you know this?
1
Not at all
2
3
4
5
Perfectly
7
Q
  1. What are the possible causes of metabolic acidosis?
A
  1. Causes of anion gap metabolic acidosis are easily remembered by pneumonic
    MUDPILES [1].
    M: methanol
    U: uremia
    D: diabetic ketoacidosis
    P: paraldehyde
    I: infection, INH therapy
    L: lactic acidosis
    E: ethanol, ethylene glycol
    S: salicylates (aspirin)
    Causes of non-gap metabolic acidosis:
    • Excessive administration of 0.9% normal saline
    • GI losses: diarrhea, ileostomy, neobladder, pancreatic fistula
    • Renal losses: renal tubular acidosis
    • Drugs: acetazolamide
How well did you know this?
1
Not at all
2
3
4
5
Perfectly
8
Q
  1. What are the possible causes of respiratory acidosis?
A
  1. Respiratory acidosis which is from increased CO2 is due either to increased pro-
    duction or decreased elimination [2].
    (a) Increased production of CO2:
    • Malignant hyperthermia
    • Hyperthyroidism
    • Sepsis
    • Overfeeding
    (b) Decreased elimination of CO2:
    • Intrinsic pulmonary disease (pneumonia, ARDS, fibrosis, edema)
    • Upper airway obstruction (laryngospasm, foreign body, OSA)
    • Lower airway obstruction (asthma, COPD)
    • Chest wall restriction (obesity, scoliosis, burns)
    • CNS depression (anesthetics, opioids, CNS lesions)
    • Decreased skeletal muscle strength (myopathy, neuropathy, residual effects of neuromuscular blocking drugs)
    • Rarely, anexhausted soda–lime or incompetent one-way valve in an anesthesia delivery system can contribute to respiratory acidosis.
How well did you know this?
1
Not at all
2
3
4
5
Perfectly
9
Q

A patient is unresponsive and taking shallow breaths in the recovery room. Arterial
blood gas shows:
pH—7.26, CO2—69, O2—54, HCO3
−—25
Questions
1. What does the blood gas show?

A
  1. The blood gas shows hypoxia (pO2 less than 60) along with respiratory acidosis
    with little metabolic compensation [1].
How well did you know this?
1
Not at all
2
3
4
5
Perfectly
10
Q
  1. What is the difference between hypoxia and hypoxemia?
A
  1. Hypoxia is a failure of the delivery of adequate amounts of oxygen to tissue. This can be local, regional, or global. Hypoxemia is a low blood oxygen content. SaO2 <90%, PaO2 <60 mmHg.
How well did you know this?
1
Not at all
2
3
4
5
Perfectly
11
Q
  1. What is the most common cause of hypoxia seen in the perioperative period?
A
  1. Hypoventilation is a common problem noted in the postoperative period. ○ There are a number of possible causes [1].
    ○ Some of the more common etiologies that might be seen in the PACU:
    (a) Poor respiratory drive—may be caused by narcotics, sedatives, and inhalational anesthetic agents.
    (b) Muscle weakness—most commonly related to residual neuromuscular
    blockade. It could also be seen in patients with neuromuscular disease.
    (c) Airway obstruction—could be secondary to residual muscle weakness, airway surgery, or laryngospasm. The patient could have a history of obstructive sleep apnea.
How well did you know this?
1
Not at all
2
3
4
5
Perfectly
12
Q

A patient is unresponsive and taking shallow breaths in the recovery room. Arterial blood gas shows: pH—7.26, CO2—69, O2—54, HCO3−—25
4. What are some other possible causes of hypoxia?

A
  1. Hypoxia can be divided [2]:
    (a) Hypoxic hypoxia—an inadequate amount of oxygen getting to the lungs [1]
    • Low inspired oxygen concentration, e.g., high altitude
    • Airway obstruction
    • Hypoventilation [3]
    • Neuromuscular disease
    • Shunting and V/Q mismatch [1, 3]
    • Interstitial lung disease
    (b) Anemic hypoxia
    • Low hemoglobin level
    • Abnormal hemoglobin, e.g., methemoglobin or carbon monoxide poison-
    ing [1]
    (c) Stagnant or circulatory hypoxia—inadequate blood flow to the tissues
    • Generalized—causes
    – Low cardiac output—heart failure, MI [3]
    – Poor cardiac venous return
    – Shock
    • Localized—causes
    – Anything which limits flow to the local tissue
    (d) Histotoxic hypoxia
    • Cells are unable to utilize oxygen, e.g., cyanide toxicity
How well did you know this?
1
Not at all
2
3
4
5
Perfectly
13
Q
  1. What are some of the physiologic effects, signs, and symptoms of hypoxia?
A
  1. Effects will vary based on the cause and what tissues are hypoxic.
    (a) Generalized hypoxia—signs and symptoms [1]
    • Tachypnea
    • Tachycardia
    • Shortness of breath
    • Sweating
    • Cyanosis (cherry red skin color in cyanide toxicity)
    • Headache
    • Confusion
    • Restlessness
    • Seizure
    • Coma
How well did you know this?
1
Not at all
2
3
4
5
Perfectly
14
Q
  1. How would you treat hypoxia?
A

○ Initial treatment is oxygen administration.
○ Further therapy may be required
depending on the cause.
Examples:
(a) Acute asthma exacerbation bronchodilators
(b) Embolus or thrombus—removal

How well did you know this?
1
Not at all
2
3
4
5
Perfectly
15
Q
  1. What is the alveolar gas equation and how might it help in identifying the cause of hypoxia?
A
  1. Alveolar gas equation [
    (a) PAO2= FiO2 ×( Patm- Pvapor) - PCO2/R
    PAO2—partial pressure of alveolar O2
    FiO2—fraction of inspired O2
    Patm—atmospheric pressure
    PH O2 —partial pressure of water vapor
    PaCO2—partial pressure CO2 in arterial blood
    R—respiratory exchange ratio, usually 0.8
    Alveolar–arterial gradient [3]
    (b) A–a gradient = PAO2 − PaO2
    PaO2—partial pressure of arterial O2
    A–a gradient may be used to help determine the cause of hypoxia. The gradient
    increases with age. Normal gradient is less than 10 mmHg plus 1 mmHg per
    decade of life.
    Hypoxia with normal A–a gradient
    • Hypoventilation
    • Low partial pressure of inspired O2 such as at high altitudes
    Hypoxia with high A–a gradient
    • Diffusion impairment in alveolus
    • V/Q mismatch
    • Right to left shunt
How well did you know this?
1
Not at all
2
3
4
5
Perfectly
16
Q

A patient with closed fracture of the lower extremity is scheduled for an ORIF. The patient is an unaccompanied, slender, 26-year-old male who cannot give a good history due to confusion and has deep, rapid breathing with a distinctive odor. His vital signs show mild hypotension, tachycardia, and low-grade fever. Investigations demonstrate Na+ 132, K+ 4.8, Cl− 92, HCO3
− 12, BUN 24 mg, creatinine 1.6 mg, Ca++ 7.8 mg, and blood sugar of 318 mg/dl. Arterial blood gas shows a pH of 7.24, PCO2 28, PO2 76, HCO3 12, BE of 14, and O2 sat of 93%. His CBC is normal with mild leukocytosis and evidence of hemoconcentration. The chest X-ray is unremarkable and EKG shows sinus tachycardia.
1. What is the likely initial diagnosis of this patient and how can you confirm the diagnosis?

A
  1. The presentation of this young patient with altered sensorium, “Kussmaul” breathing, hyperglycemia, and metabolic acidosis strongly suggests diabetic
    ketoacidosis (DKA). The diagnosis can be confirmed by the presence of ketone bodies in the urine and serum . Concomitant lactic acidosis must also be investigated ].
    ○ As with any patient with a traumatic injury and altered sensorium, radiological testing for cervical spine and cranial pathology must be done.
How well did you know this?
1
Not at all
2
3
4
5
Perfectly
17
Q
  1. What are abnormal laboratory values in the BMP and ABGs that are seen in this
    condition DKA?
A
  1. The laboratory values in DKA will show evidence of metabolic acidosis, electrolyte derangements, and evidence of severe dehydration.
    (a) BMP
    • Na+—there is a total body loss of Na+; the levels can be low normal.
    Correction must be made for undermeasurement of Na+ due to hyperglycemia (add 1.6 meq/L to the measured Na+ for every 100 mg of glucose above 100 mg/dl level).
    • K+—there can be a significant total body loss of 3–10 meq/kg of K+. The initial serum K+ level may be paradoxically high due to both volume
    contraction and decreased movement into the intracellular compartment
    ].
    • Cl−—will be decreased.
    • HCO3 will be decreased.
    • Anion gap—will be increased above normal 10–14 meq/L . This gap is calculated by the formula:
    AG = Na+ − (Cl− + HCO3
    −)
    • BUN—will be increased.
    • Creatinine—may be mildly increased.
    • Ca++—may be decreased. Additionally magnesium and phosphate depletion can also occur.
    • Glucose—increases to levels greater than 250–600 mg/dl [4] but rarely may be normal, when called euglycemic DKA .
    (b) ABG
    • pH—usually less than 7.3
    • PaCO2—usually lower due to respiratory compensation for metabolic acidosis
    • PaO2—usually low normal unless a pneumonic process causes it to be
    low
    • HCO3—will be lower due to metabolic acidosis
    • BE—will be lower to indicate significant metabolic acidosis
    • O2 saturation—will be in the low 90 s with O2 supplementation unless a pneumonic process causes it to be loweryd
How well did you know this?
1
Not at all
2
3
4
5
Perfectly
18
Q

A patient with closed fracture of the lower extremity is scheduled for an ORIF.The patient is an unaccompanied, slender, 26-year-old male who cannot give a good history due to confusion and has deep, rapid breathing with a distinctive odor. His vital signs show mild hypotension, tachycardia, and low-grade fever. Investigations demonstrate Na+ 132, K+ 4.8, Cl− 92, HCO3− 12, BUN 24mg, creatinine 1.6mg, Ca++ 7.8mg, and blood sugar of 318mg/dl. Arterial blood gas shows a pH of 7.24, PCO2 28, PO2 76, HCO3 12, BE of 14, and O2 sat of 93%. His CBC is normal with mild leukocytosis and evidence of hemoconcentration. The chest X-ray is unremarkable and EKG shows sinus tachycardia.
3. What is the major differential diagnosis in this clinical condition? DKA

A
  1. ○ The major differential diagnosis in this scenario would be non-ketotic hyperosmolar hyperglycemia (NHH).
    ° In this condition the patient is generally a type 2 diabetic and as such would likely be an older and often overweight patient.
    ° The patient can present with altered mentation or in a coma.
    ° The blood sugar levels are frequently
    higher (>600 mg/dl) and there is no ketone body formation [4].
    ° Therefore metabolic acidosis if present would likely be due to the precipitant cause such as infection with lactic acidosis. ° The reason for the absence of ketone bodies is due to the presence of some circulating insulin. This insulin can prevent the alteration in fatty acid metabolism leading to ketosis but due to peripheral insulin resistance still leads to very high
    serum glucose levels.
    ° The presence of increased insulin counter regulatory hormones (esp. glucagon) exacerbates the hyperglycemia due to increased hepatic gluconeogenesis.
    ° The resultant osmotic diuresis leads to the severe dehydration (~12 L loss), azotemia, and hyperosmolarity (>330 mOsm/L) [4].
    ° Serum osmolarity is calculated by the formula 2(Na+ + K+) + Glucose/18 + B
    UN/2.8.
    ° The precipitating causes can be infection, stoppage of medication, newly diagnosed diabetes, stroke, MI, subdural hematoma, and GI diseases.
    ° The treatment of this condition is hydration, correction of electrolyte aberrations, and treatment of the causative process.
    ° Insulin use will be needed to gradually bring down the blood sugar.
How well did you know this?
1
Not at all
2
3
4
5
Perfectly
19
Q
  1. What are the principles in the treatment of this condition?DKA
A
  1. The principles for treatment of DKA are
    (a) Insulin therapy to decrease hyperglycemia and stop production of ketone bodies.
    (b) Hydration with isotonic solutions. Deficit may be up to 9 L in the average
    adult.
    ° Start with saline and convert to isotonic fluids with K+ when K+ levels start to decrease, and urine output is maintained.
    ° Change to hypotonic solution if Na+ level >150 meq/L.
    ° Bicarb therapy is only reserved for severe acidosis (pH < 7.1).
    (c) Replacement of other specific electrolytes Ca++, Mg++, PO4.
    (d) Treatment of precipitating cause—infections, interruption of insulin, MI,
    trauma, stress.
    (e) Mental status changes—may need to have airway protected and ventilator
    assistance.
    (f) Ileus and other GI presentations, e.g., acute cholecystitis, either due to systemic ketosis or incidental, must be clinically managed.
How well did you know this?
1
Not at all
2
3
4
5
Perfectly
20
Q

A patient with closed fracture of the lower extremity is scheduled for an ORIF.The patient is an unaccompanied, slender, 26-year-old male who cannot give a good history due to confusion and has deep, rapid breathing with a distinctive odor. His vital signs show mild hypotension, tachycardia, and low-grade fever. Investigations demonstrate Na+ 132, K+ 4.8, Cl− 92, HCO3− 12, BUN 24mg, creatinine 1.6mg, Ca++ 7.8mg, and blood sugar of 318mg/dl. Arterial blood gas shows a pH of 7.24, PCO2 28, PO2 76, HCO3 12, BE of 14, and O2 sat of 93%. His CBC is normal with mild leukocytosis and evidence of hemoconcentration. The chest X-ray is unremarkable and EKG shows sinus tachycardia.
5. How do the results of the BMP and ABG trend during the treatment of this
condition?

A
  1. The trending changes for electrolytes and the ABG with treatment will be:
    (a) BMP
    • Na+—should be in the upper normal range.
    • K+—after initial fluid resuscitation with use of NS (first 4 h), the K+ levels
    will drop associated with the intracellular migration due now to the pres-
    ence of insulin. K+ can be added to IV fluids once the level goes below
    4 meq/L, and a steady urine output is maintained.
    • Cl−—will increase with use of normal saline (NS). Excessive use of NS
    can lead to hyperchloremic acidosis.
    • HCO3—use of replacement NaHCO3 is not required unless acidosis is
    severe (<pH7.1).
    • Anion gap—will move toward normal gap of <11 meq/L.
    • BUN—azotemia, if present, will normalize with hydration and increased
    urine production.
    • Creatinine—as volume status and GFR improves, it should normalize
    unless kidneys are affected.
    • Ca++—can be low due to loss from osmotic diuresis—careful augmenta-
    tion along with associated Mg++ and phosphate supplementation for their
    measured deficiencies.
    • Glucose—the target is to gradually bring the blood sugar (BS) level down
    ~75–100 mg/h using regular insulin as an IV bolus (0.1 u/kg) followed by
    continuous infusion IV (0.1 u/kg/h) [4]. Rates of insulin infusion can be
    progressively ramped up with use of any standard protocol. Once BS
    levels reach the lower 200 s/dl, then 5% glucose should be added to the
    IV fluids to prevent hypoglycemia [10]. Target blood sugar is in the range
    120–150 mg/dl
How well did you know this?
1
Not at all
2
3
4
5
Perfectly
21
Q
  1. How will you continue management of this patient with the planned surgery? DKA
A
  1. Once the patient has had definitive treatment for DKA and has shown metabolic stabilization, surgery can proceed. The principles for perioperative management would include:
    (a) Continuing the use of appropriate fluids and electrolyte and IV insulin
    administration by infusion.
    (b) Precautions for a full stomach before induction if not already intubated.
    (c) Type 1 diabetics can have a difficult airway due to stiffening of tissues of the upper airway and rigidity of the cervical spine.
    (d) Arterial line and good venous access for this particular case would be appropriate. Central venous access for volume estimation in major surgery or in patients with comorbidity would be appropriate.
    (e) Glucose checks at least hourly under anesthesia with BMP and ABG at regular intervals.
    (f) At the end of the procedure, extubation would depend on preinduction status, intraoperative course, and emergence profile. The postoperative care should continue in an ICU setting with treatment for both initiating and
    coexisting clinical issues.
    (g) Once stable, the diet and treatment plan must be made with type, amount, and route of administration of insulin determined.
How well did you know this?
1
Not at all
2
3
4
5
Perfectly
22
Q

Below are the values obtained on arterial blood gas measurement of a patient on
cardiopulmonary bypass (CPB)
pH 7.44
pCO2 30.8 mmHg
pO2 354 mmHg
BE 3 mmol/L
HCO3 27 mmol/L
SpO2 100%
Sample type: arterial
FiO2: 35
Temp: 30°C
1. What type of clinical test is this and what does it measure?

A

○ This is an arterial blood gas (ABG) analysis; it gives information about the adequacy of a patient’s gas exchange and acid–base status.
○ It is used perioperatively, during CPB and also in severe lung disease (severe asthma in the ER), cardiac and kidney failure, uncontrolled diabetes, severe infections, drug overdose, and also in the ICU.
○ An abnormal pH value as in acidosis or alkalosis can occur in disease states.
○ ABG helps us to determine if the acid–base derangement is respiratory or metabolic in origin.
○ The result is always reported taking into consideration the temperature of the patient at the time of collection.

How well did you know this?
1
Not at all
2
3
4
5
Perfectly
23
Q
  1. What is the importance of temperature in the reported result?Blood gas measurement
A
  1. The arterial blood sample is preheated to 37°C prior to measurement. If the actual patient temperature is keyed in, modern blood gas machines will report the pH value for that temperature as well.
    ○ This is calculated mathematically from the pH measured at 37°C.
    ○ For clinical use, the Rosenthal correction factor is recommended and is done as follows:
    Change in pH = 0.015 pH units per degree Celsius change in temperature.
    ○ According to Henry’s law, the solubility of a gas increases with decrease in temperature. PO2 is 5 mmHg lower and PCO2 is 2 mmHg lower for each degree below 37°.
    ○ Hypothermia causes a decrease in the PCO2 (hypocarbia) and a concomitant increase in the pH (alkalemia), yet the total body CO2 content remains the same.
    ○ There are two blood gas management strategies in hypothermia—temperature correction (pH stat) or not (α stat).
    ○ These have different effects on cerebral blood flow, oxygen dissociation curve, and intracellular enzyme and protein activity.
How well did you know this?
1
Not at all
2
3
4
5
Perfectly
24
Q
  1. What is the pH-stat approach?
A
  1. In the pH-stat strategy (in hypothermic CPB or deep hypothermic circulatory arrest [DHCA]), blood gases are corrected to patient’s temperature by decreasing
    the CPB gas sweep rate (which decreases the removal of CO2) or adding CO2 to the oxygenator to maintain a constant pH of 7.4 and PCO2 of 40 mmHg at varying patient temperature.
    ○ pH stat requires an increased total body CO2 content to maintain neutrality during hypothermia thereby producing an acidotic state.
    ○ The increased PCO2 exerts a cerebral vasodilatory effect (loss of autoregulation).
    ○ Proposed benefits of pH stat include rightward shift of the oxyhemoglobin dissociation curve increasing oxygen delivery, increased cerebral blood flow (CBF) decreasing the risk of cerebral ischemia during CPB, more complete and
    faster cooling, and greater suppression of cerebral metabolic rate
How well did you know this?
1
Not at all
2
3
4
5
Perfectly
25
Q
  1. What is the α-stat approach?
A

In the α-stat approach, there is no temperature correction; blood gases are always
interpreted at the same normal (37°C) temperature irrespective of the actual
patient temperature. Neutrality is maintained only at 37°C permitting the hypo-
thermic alkaline drift. No CO2 is added and cerebral autoregulation is
maintained.
Alpha is the ratio of protonated to total imidazole of histidine (degree of dis-
sociation) residues among protein molecules at 37°C. At the normal intracellular
pH of 6.8, it is 0.55. The alpha value remains constant despite changes in temperature as the pK (dissociation constant) changes with temperature. This is opti-
mal for intracellular enzyme structure and function which is the reason cited by
its proponents who also argue that the increased CBF with the pH stat strategy
may put the brain at risk from microemboli or cerebral edema. They also argue
that the alkaline pH in the α-stat approach is beneficial before the ischemic insult
of circulatory arrest

How well did you know this?
1
Not at all
2
3
4
5
Perfectly
26
Q
  1. Which is better? Alpha vs beta stat
A

The debate over the optimal blood gas management is not over.
This may not be important in moderate hypothermia but may be critical in deep hypothermia.
○ In adults α-stat strategy is preferred to maintain cerebral autoregulation and limit
cerebral embolic load, and in neonates and children, the pH-stat strategy demonstrated better outcomes.
○ The reason for the difference may be related to the differences in the mechanism of brain injury on CPB.
○ In children, due to the aortopulmonary collaterals causing hypoperfusion, the pH-stat strategy with its increased CBF seemed to provide benefit [6].

How well did you know this?
1
Not at all
2
3
4
5
Perfectly
27
Q

You are asked to see a healthy female at 38-weeks gestation. She has the following
lab results:
Complete blood count (CBC)
• White blood count (WBC)—12,800 × 103/mm3
• Hemoglobin (Hgb)—9.5 g/dL
• Hematocrit (Hct)—28.5%
• Platelets—148 × 109/L
Chemistries
• Sodium (Na)—136 meq/L
• Potassium (K)—3.9 meq/L
• Chloride (Cl)—108 meq/L
• Bicarbonate (HCO3)—21 mmol/L
• Anion gap (AG)—7 mmol/L
• Blood urea nitrogen (BUN)—6 mg/dL
• Creatinine (Cr)—0.6 mg/dL
• Glucose—91 mg/dL
• Total protein—5.8 g/dL
• Albumin—3.2 g/dL
• Calcium (Ca)—8.7 mg/dL
• Total bilirubin—0.4 mg/dL
• Aspartate transaminase (AST/SGOT)—20 U/L
• Alanine transaminase (ALT/SGPT)—12 U/L
• Alkaline phosphatase (AP)—165 U/L
1. What is the upper limit of normal for a WBC count in a term patient?

A

○ The upper limit for WBC increases through pregnancy.
○ In the third trimester, this
reaches 16,900/mm3. This is primarily from an increase in neutrophils [1].
○ There is frequently a spike in labor.

How well did you know this?
1
Not at all
2
3
4
5
Perfectly
28
Q
  1. In a term patient what is the normal hemoglobin range? What level is considered to be anemia?
A

○ The normal hemoglobin range during the third trimester is 9.5–15 gm/dL
○ Anemia in pregnancy is defined as a Hgb below 11 gm/dL (compared to a threshold of below 12 gm/dL for the non-parturient) by the American College of Obstetrics and Gynecology and the World Health Organization
○ The most common cause of anemia in pregnancy is iron deficiency. Other causes include micronutrient deficiencies, chronic inflammation, and inherited disorders such as sickle cell and the thalassemias.
○ The increase in blood volume in pregnancy results in a relatively lower Hct when compared with nonpregnant females. This is because the plasma volume increases at a higher percentage than does the red cell mass.

How well did you know this?
1
Not at all
2
3
4
5
Perfectly
29
Q
  1. What is the normal lower limit of a platelet count in pregnancy?
A

○ Platelet count normal range changes very little in pregnancy. This range is 146–429 × 109/L near term.
○ Approximately 8% of pregnant patients at term will have platelet counts <150,000 and in about 1% it will be <100,000

How well did you know this?
1
Not at all
2
3
4
5
Perfectly
30
Q
  1. How does pregnancy affect the serum bicarbonate level?
A

Bicarbonate levels are decreased throughout pregnancy [1].
○ Tidal volume increases by about 1/3, and the respiratory rate increases slightly resulting in a 30–50% increase in minute ventilation.
○ The CO2 decreases to approximately 30 mmHg.
○ Metabolic compensation results in a bicarbonate level of about 20 meq/L

How well did you know this?
1
Not at all
2
3
4
5
Perfectly
31
Q
  1. How do the renal function tests change BUN and creatinine in pregnancy?
A

Both levels are decreased because of an increase by 50% in the glomerular filtration rate (GFR) and the increase in creatinine clearance from 120 ml/min to greater that 150 ml/min

How well did you know this?
1
Not at all
2
3
4
5
Perfectly
32
Q
  1. Are the plasma proteins affected by pregnancy?
A

Total plasma proteins and albumin are both decreased.

How well did you know this?
1
Not at all
2
3
4
5
Perfectly
33
Q
  1. Which liver function test is frequently affected in pregnancy?
A

Alkaline phosphatase (AP) is commonly increased 2–4 times above nonpregnant values because of production by the placenta

How well did you know this?
1
Not at all
2
3
4
5
Perfectly
34
Q

A 27-year-old G4P3 presented to antepartum clinic with high blood pressure and
epigastric pain. On physical examination the patient had mild epigastric tenderness
and 2+ edema over both lower extremities.
Vital signs: BP 170/120 mmHg, HR 90 bpm, RR 20 bpminute, SpO2 95% on
room air
Hb 11 mg/dL
Hct 33
Platelets 90 K
Creatinine >1.2 mg/dL
Billirubin >1.2 mg/mL
Uric acid >6 mg/mL
LDH >600 IU/L
Elevated AST/ALT
Proteinuria >0.3 g in a 24 h urine specimen
1. What laboratory work-up is needed to confirm your diagnosis?

A

Complete blood cell count (CBC), serum electrolytes, blood urea nitrogen, creatinine, liver function test, serum uric acid, urine analysis—microscopic and 24 h specimen for protein and creatinine clearance. According to the American
Congress of Obstetricians and Gynecologists (ACOG) practice bulletin in 2002, preeclampsia is defined as the new onset of hypertension and proteinuria after 20 weeks’ gestation [1]. Proteinuria is a key factor in order to differentiate preeclampsia vs gestational hypertension and chronic hypertension in pregnancy. However in 2013 ACOG guidelines, proteinuria was removed from the diagnostic criteria of preeclampsia as it is nonspecific and doesn’t always correlate with maternal and
fetal outcomes. ACOG has suggested that any parturient with new-onset hyperten-sion at 20 weeks ofpregnancy or beyond, along with either of the following conditions, should be diagnosed with preeclampsia even in the absence of proteinuria.
(a) Reduced platelet counts
(b) Renal insufficiency
(c) Severe headache
(d) Cardiopulmonary compromise
(e) Impaired liver function2. E Neurogenic shock

How well did you know this?
1
Not at all
2
3
4
5
Perfectly
35
Q
  1. How will you differentiate mild vs severe forms of the condition based on proteinuria?
A

Mild preeclampsia: BP ≥140/90 mmHg after 20 weeks of gestation
(a) Proteinuria 300 mg/24 h or 1+ result on urine dipstick
Severe preeclampsia: BP ≥160/110 mmHg
(b) Proteinuria >5 g/24 h
New 2020 guidelines

How well did you know this?
1
Not at all
2
3
4
5
Perfectly
36
Q

What is important to look for in the complete blood count (CBC) in pregnantwomen?

A

Thrombocytopenia is present in 15–30% of women with preeclampsia, and it is the most common hematologic abnormality.
○ Platelet counts of less than 100,000/mm3 occur mostly in severe preeclampsia or HELLP syndrome.
○ Platelet counts also correlate with the severity of the disease process and the incidence of placental abruption. Therefore, serial CBC (6 h apart) should be drawn in a patient with severe preeclampsia to follow the progression of the disease.
○ Women with preeclampsia are usually intravascular volume depleted which causes hemoconcentration with false elevation of Hb and Hct. It is also an indicator of severity, although measurements are decreased if hemolysis is present with HELLP syndrome.

How well did you know this?
1
Not at all
2
3
4
5
Perfectly
37
Q

How are blood urea nitrogen (BUN), creatinine, and uric acid levels affected in this condition?

A

○ Glomerular filtration rate (GFR) increases by 40–60% during the first trimester of pregnancy which causes a decrease in levels of BUN, creatinine, and uric acid.
○ These are the serum markers of renal clearance. In preeclampsia, GFR is 34% lower than in normal pregnancy. Decrease in GFR contributes to higher BUN and creatinine levels in women with preeclampsia.
○ Abnormal or rising creatinine level suggests severe preeclampsia, especially in the presence of oliguria.
○ Urate clearance decreases in women with preeclampsia with resulting increase in serum uric acid concentration which is possibly an early indicator of preeclampsia. Serum urate greater than 5.5 mg/dL is diagnostic of preeclampsia.

How well did you know this?
1
Not at all
2
3
4
5
Perfectly
38
Q
  1. Is the epigastric pain significant in this patient with severe preeclampsia?
A

○ Epigastric or subcostal pain is an ominous symptom and is usually caused by the distension of the liver capsule by edema or subcapsular hemorrhage.
○ Hepatic dysfunction is frequently seen manifested as an increase in serum transaminase levels in patients with preeclampsia which should be followed serially to assess the disease progression to HELLP syndrome, if it occurs.

How well did you know this?
1
Not at all
2
3
4
5
Perfectly
39
Q
  1. What is HELLP syndrome and what are some of the diagnostic criteria?
A

HELLP syndrome is a variant of severe preeclampsia characterized by hemolysis, elevated liver enzymes, and low platelet counts. It is associated with rapid
clinical deterioration.
(a) Diagnostic criteria:
• Hemolysis:
– Bilirubin >1.2 mg/dL
– Lactic dehydrogenase >600 IU/L
– Abnormal peripheral blood smear
(b) Elevated liver enzymes:
• Serum glutamic oxaloacetic transaminase (SGOT) ≥70 IU/L
• aspartate aminotransferase (AST) and alanine aminotransferase (ALT) elevated more than twice the upper limit of normal,
(c) Low platelet counts:
• <100,000/mm3
Hemolysis is usually reflected as microangiopathic hemolytic anemia on peripheral blood smear which demonstrates schistocytes, burr cells, and echinocytes.

How well did you know this?
1
Not at all
2
3
4
5
Perfectly
40
Q
  1. What will you look for in the DIC panel?
A

○ Patients with severe preeclampsia and HELLP syndrome can develop disseminated intravascular coagulation, and its presence should be confirmed by laboratory work-up.
○ Blood work will show a decrease in fibrinogen level and severe thrombocytopenia as these procoagulants are decreased in DIC along with an increase in D-dimer level and fibrinogen degradation product (FDP)

How well did you know this?
1
Not at all
2
3
4
5
Perfectly
41
Q

A 27-year-old woman, G1P0, had an emergent cesarean section with an epidural
anesthesia for severe fetal heart rate deceleration. A nuchal cord was found at the
time of delivery by the obstetrician and an umbilical blood gas was ordered.
Umbilical artery blood gas values were as follows:
pH 7.27, PCO2 50 mmHg, pO2 20 mmHg, HCO3 23 mEq/L, Base excess
−3.6 mEq/L
1. How will you interpret this blood gas value and what are the different types of
acidosis?

A

The given values are representative of a normal blood gas for a newborn.
○ The table below lists normal findings for a fetal blood gas at term gestation
During oxidative metabolism, carbonic acid is produced, which is usually cleared by the placenta as carbon dioxide [2].
○ If placental blood flow is not adequate, then CO2 elimination can be affected leading to respiratory acidosis.
○ Lactic and beta-hydroxybutyric acids are produced as a result of anaerobic ,metabolism , which requires hours of metabolic clearance and contributes to metabolic and mixed acidosis.

How well did you know this?
1
Not at all
2
3
4
5
Perfectly
42
Q
  1. What are the different methods to assess fetal acid–base balance?
A

Fetal acid–base balance can be accessed via a number of ways:
(a) Antepartum: by percutaneous umbilical cord blood sampling
(b) Intrapartum: by fetal scalp blood sampling (after membranes have
ruptured)
(c) Postpartum: by umbilical cord blood sampling

How well did you know this?
1
Not at all
2
3
4
5
Perfectly
43
Q

In the newborn, is blood sampling for blood gas analysis performed from the umbilical artery or vein?

A

○ Usually, blood samples from both umbilical artery and vein are collected, which represent the fetal and maternal condition, respectively. ○ In addition to maternal condition, umbilical vein blood samples also represent the utero-placental gas
exchange.
○ In order for blood samples to be accurate, the umbilical cord should be double clamped at least 10–20 cm apart immediately after delivery, and the blood samples should be drawn via heparinized syringe within 15 min of delivery .
○ For accuracy, the samples should be analyzed within 30–60 min.
○ Air bubbles should also be removed from the syringe to get accurate pO2 measurement.
○ In low birth weight infant, it can be difficult to obtain blood sample from the umbilical artery, especially if it is small. In such situations, the newborn should be carefully evaluated for arterial academia, since isolated venous blood gas pH can be normal.

How well did you know this?
1
Not at all
2
3
4
5
Perfectly
44
Q

Is fetal blood gas estimation more reliable than Apgar scores in assessing a newborn’s condition?

A

○ Umbilical cord blood gas analysis is routinely ordered by obstetricians if there is suspicion of neonatal depression.
○ It reflects the fetal condition immediately before delivery and is a more objective indication of a newborn’s condition than Apgar score, as Apgar score is usually done after the delivery at 1 min, 5 min, and 10 min interval.
○ However, there is usually a time lag between blood gas sampling and analysis.
○ In the meantime neonatal condition should be assessed by the Apgar score.
○ Another factor that can affect umbilical arterial blood pH is the mode of delivery.
○ A fetus that is delivered via spontaneous vaginal delivery will have a lower pH than the one delivered by elective cesarean section as the former has to go through the stress of labor.
○ Duration of labor can also affect pH measurement, as prolonged labor in nulliparous women will lower the fetal pH.

How well did you know this?
1
Not at all
2
3
4
5
Perfectly
45
Q

Is umbilical blood gas analysis done for every newborn?

A

In 2006, the American Congress of Obstetricians and Gynecologists (ACOG)
recommended cord blood gas for:
(a) Cesarean delivery for fetal compromise
(b) Low 5-min Apgar score
(c) Severe growth restriction
(d) Abnormal FHR tracing
(e) Maternal thyroid disease
(f) Intrapartum fever
(g) Multiple gestation

How well did you know this?
1
Not at all
2
3
4
5
Perfectly
46
Q

What is the implication of fetal blood gas acidosis?

A

○ The type of acidosis, if present, should be ascertained, as metabolic and mixed acidosis are associated with an increased incidence of neonatal complications and death.
○ One study found a higher incidence of neonatal death when the pH of umbilical arterial blood was less than 7.00. Seizures were also reported in infants with pH of less than 7.05.

How well did you know this?
1
Not at all
2
3
4
5
Perfectly
47
Q

Does fetal acidosis have long-term sequelae on neonatal outcome?

A

○ According to the ACOG Task Force in 2006, an umbilical artery pH of less than 7.0 and a base deficit of greater than or equal to 12 mmol/L at delivery pointed toward an acute intrapartum hypoxic event which could eventually cause cerebral palsy.
○ Whenever pH is less than 7.00, the base deficit and bicarbonate values are the predictors for neonatal morbidity.
○ Moderate to severe complications occur in 10% of infants when base deficit is 12–16 mmol/L, which increases to 40% when base deficit is more than 16 mmol/L.

How well did you know this?
1
Not at all
2
3
4
5
Perfectly
48
Q

What should you do as an anesthesiologist during routine/urgent cesarean
section to improve fetal outcome?

A

○ There are certain things that can be done by an anesthesiologist to improve the fetal outcome during routine/urgent c-section to maintain adequate placental perfusion.
(a) Provide left uterine displacement to avoid aorto–caval compression by
gravid uterus
(b) Support the hemodynamics by intravenous administration of fluids and vasopressors if needed to maintain utero-placental circulation (as it is MAP dependent)
(c) If general anesthesia is chosen, then maintain proper oxygenation by providing at least 50% oxygen when mixed with 50% N2O to avoid hypoxia

How well did you know this?
1
Not at all
2
3
4
5
Perfectly
49
Q

A 25-year-old man is brought to the emergency department after a motor vehicle
accident in which he was an unrestrained passenger. He is otherwise healthy.
Clinically he was alert but confused and in pain. BP on arrival was 88/60 mmHg, HR
124/min, and RR 24/min. He weighed 70 kg. His skin was cool and clammy to touch.
X-rays showed right thigh and pelvic fracture. CT scans of the head, chest, and abdo-
men were normal. CT scan of the pelvis showed a complex fracture of pelvis.
Labs on admission were hemoglobin 9.1 gm/dL, platelets 118,000/mL, pro-
thrombin time (PT) and partial thromboplastin time (PTT) mildly elevated, and lac-
tate 4.2 mmol/L. The patient had received 1500 cc of normal saline from the time of
injury to admission.
Questions
1. Is the patient in hemorrhagic shock?

A

○ The patient is in hemorrhagic shock, a condition produced by rapid and significant loss of intravascular volume, which may lead sequentially to hemodynamic instability and decreased tissue perfusion.
○ The injuries this patient suffered are associated with significant amount of bleeding.
○ Fractures of the pelvis and femurs can hide massive amounts of bleeding with little external evidence and potentially put the patient at risk for hemorrhagic shock.
○ Signs of shock in this patient are decrease in BP, tachycardia, tachypnea, confusion, cool and clammy skin, and elevated lactate.
○ Other signs that could be present in shock state include oliguria and metabolic acidosis.
○ This patient most likely has class III hemorrhagic shock

How well did you know this?
1
Not at all
2
3
4
5
Perfectly
50
Q

What is the estimated blood volume in this patient? 70kg man

A

○ The average adult blood volume represents 7% of body weight (or 70 mL/kg of
body weight).
○ Estimated blood volume for a 70 kg person is approximately 5 L.

How well did you know this?
1
Not at all
2
3
4
5
Perfectly
51
Q

A 25-year-old man is brought to the emergency department after a motor vehicle accident in which he was an unrestrained passenger. He is otherwise healthy. Clinically he was alert but confused and in pain. BP on arrival was 88/60mmHg, HR 124/min, and RR 24/min. He weighed 70kg. His skin was cool and clammy to touch. X-rays showed right thigh and pelvic fracture. CT scans of the head, chest, and abdomen were normal. CT scan of the pelvis showed a complex fracture of pelvis. Labs on admission were hemoglobin 9.1 gm/dL, platelets 118,000/mL, prothrombin time (PT) and partial thromboplastin time (PTT) mildly elevated, and lactate 4.2mmol/L.The patient had received 1500cc of normal saline from the time of injury to admission.
Does this patient need blood transfusion with hemoglobin of 9.1 gm/dL?

A

Yes, maintaining a higher hemoglobin level of 10 g/dL is a reasonable goal in actively bleeding patients and with signs of shock.
○ Hemoglobin concentration in an actively bleeding individual has dubious diagnostic value because it takes time for the various intravascular compartments to equilibrate.
○ Hemoglobin concentration should not be the only therapeutic guide for blood transfusion in actively bleeding patients.
○ Rather, therapy should be guided by the rate of bleeding and changes in hemodynamic parameters.

How well did you know this?
1
Not at all
2
3
4
5
Perfectly
52
Q

A 25-year-old man is brought to the emergency department after a motor vehicle accident in which he was an unrestrained passenger. He is otherwise healthy.
Clinically he was alert but confused and in pain. BP on arrival was 88/60 mmHg, HR 124/min, and RR 24/min. He weighed 70 kg. His skin was cool and clammy to touch.
X-rays showed right thigh and pelvic fracture. CT scans of the head, chest, and abdomen were normal. CT scan of the pelvis showed a complex fracture of pelvis.
Labs on admission were hemoglobin 9.1 gm/dL, platelets 118,000/mL, pro-
thrombin time (PT) and partial thromboplastin time (PTT) mildly elevated, and lactate 4.2 mmol/L. The patient had received 1500 cc of normal saline from the time of injury to admission
Was this patient coagulopathic on admission?

A

○ Yes, as evident by the prolongation of the PT and PTT.
○ Many patients with severe injuries seen in the emergency department have an established coagulopathy called trauma-induced coagulopathy or acute traumatic coagulopathy (ATC).
○ ATC is an impairment of hemostasis and activation of fibrinolysis that occurs early after injury and is biochemically evident prior to, and independent of, the development of significant acidosis, hypothermia, or hemodilution.
○ This has driven a change in the early resuscitation of these patients with blood and blood components

53
Q

How is ATC initiated?

A

○ Acute traumatic coagulopathy (ATC) or trauma-induced coagulopathy (TIC), is an impairment of hemostasis and activation of fibrinolysis that occurs in response to severe injury and hypoperfusion.
○ TIC can contribute significantly to the bleeding from the injury.
○ It is mediated primarily by activation of the thrombomodulin-protein C system.
○ Activated protein C inhibits coagulation cofactors V and VIII, reducing further thrombin generation.
○ Platelet dysfunction has an important role in the pathophysiology of TIC.
○ When present in excess, activated protein C depletes plasminogen activator inhibitor-1 (PAI-1) thus reducing tissue plasminogen activator (tPA) inhibition and accelerating the conversion of plasminogen to plasmin

54
Q

What other factors exacerbate ATC?

A

Acidosis and hypothermia alongside consumption and blood loss and the dilutional effects of resuscitation may worsen ATC/TIC.

55
Q

What are the current guidelines regarding transfusion of blood and blood products in massive trauma?

A

○ The recommendations suggest early plasma-based resuscitation, targeting ratios of packed red blood cells, FFP, and platelets approaching 1:1:1.
○ The PROPPR
(Pragmatic Randomized Optimal Platelet and Plasma Ratios) trial randomly assigned 680 severely injured patients identified at risk of requiring massive transfusion from 12 North American level I trauma centers to transfusions of plasma, platelets, and red blood cells in ratios of either 1:1:1 or 1:1:2.
○ There were no significant differences in primary outcomes of 24-h or 30-day mortality between the groups. But, death from hemorrhage was significantly less common in the 1:1:1 cohort at 3 h after injury

56
Q

HR 105, BP 155/89, Sats 94%.
Hb 9.9 g/dL, Hct 29.7%, Na 120 mEq/L, K 3.2 mEq/L, Ca 9.1 mg/dL, BUN
7 mg/dL, glucose 100 mg/dL.
Above values are obtained in a patient in PACU 30 min post-TURBT. The patient
is extremely drowsy and not arousable. No medications were given in PACU.
Questions
1. What are the causes of postoperative drowsiness and the likely cause in this
patient?

A

(a) Medications including long-acting anesthetics, benzodiazepines, barbitu-
rates, long-acting opiates, or large doses of fentanyl
(b) Timing of the medications—if the medications were given toward the end of
the procedure
(c) Profound hypoxemia
(d) hypercarbia—PCO2 greater than 75 mmHg
(e) Hypothermia and hypotension
(f) Hypoglycemia and hyperglycemia
(g) Cerebral—ischemia, hemorrhage, preexisting causes like tumor, trauma, sei-
zures, and intracranial spread of local anesthetics (associated with apnea)
(h) TURP syndrome—circulatory overload, hyponatremia, glycine, and ammonia toxicity
(i) Hypothyroidism, hepatic or renal failure
Likely cause in this patient is hyponatremia (defined as serum sodium less than
135 mEq/L).

57
Q

HR 105, BP 155/89, Sats 94%.
Hb 9.9 g/dL, Hct 29.7%, Na 120 mEq/L, K 3.2 mEq/L, Ca 9.1 mg/dL, BUN
7 mg/dL, glucose 100 mg/dL.
Above values are obtained in a patient in PACU 30 min post-TURBT. The patient
is extremely drowsy and not arousable. No medications were given in PACU
2. What is the next step?

A

○ At this stage a few things need to be done—clinical assessment of the patient for acuteness (hyponatremia 120 mmol/L and below is severe), volume status, calculation, and measurement of plasma osmolality and measurement of urine
sodium to assess the cause of hyponatremia
○ Plasma osmolality is calculated with the formula:
(2 × Na + K) + (glucose/18) + (urea/2.8)
○ Lab values for glucose and urea are reported in conventional units (mg/dL) and need conversion to SI units (mmol/L). An alternate method of conversion is glucose × 0.0555 and urea × 0.357.
Normal = 280–300 mosm/kg.
○ The difference between the measured and calculated osmolality is the osmolal gap normally less than 10 mosm/kg.
○ Elevated levels are seen in the presence of ethanol, methanol, isopropyl alcohol, and ethylene glycol

58
Q

HR 105, BP 155/89, Sats 94%.
Hb 9.9 g/dL, Hct 29.7%, Na 120 mEq/L, K 3.2 mEq/L, Ca 9.1 mg/dL, BUN
7 mg/dL, glucose 100 mg/dL.
Above values are obtained in a patient in PACU 30 min post-TURBT. The patient
is extremely drowsy and not arousable. No medications were given in PACU
3. How do you calculate plasma osmolality?

A

○ Osmolarity is a measure of the osmoles of solute per liter of solution.
○ A capital letter M is used to designate units of mmol/L. Volume of solution changes with the amount of solute added and also with temperature and pressure changes.
○ Osmolality is a measure of the osmoles of solute per kilogram of solvent and
is reported commercially using mOsm/kg. ○ As the amount of solvent remains
constant regardless of temperature and pressure changes, osmolality is preferred
and is commonly used.
○ Plasma osmolality is calculated with the formula:
(2×Na+K)+(glucose/18)+(urea/2.8) Lab values for glucose and urea are reported in conventional units (mg/dL) and need conversion to SI units (mmol/L).
○ An alternate method of conversion is glucose × 0.0555 and urea × 0.357. ○ Normal=280–300 mosm/kg.
○ The difference between the measured and calculated osmolality is the osmolal gap normally less than 10 mosm/kg.
○ Elevated levels are seen in the presence of ethanol, methanol, isopropyl alcohol, and ethylene glycol.

59
Q

What is the difference between osmolality and osmolarity?

A

Osmolarity is a measure of the osmoles of solute per liter of solution. A capital letter M is used to designate units of mmol/L.Volume of solution changes with the amount of solute added and also with temperature and pressure changes. Osmolality is a measure of the osmoles of solute per kilogram of solvent and is reported commercially using mOsm/kg. As the amount of solvent remains constant regardless of temperature and pressure changes, osmolality is preferred and is commonly used.

Management [1, 3, 4]:
(a) Correct underlying cause.
(b) If severe, expert help should be obtained.
(c) The magnitude and rapidity of correction is controversial due to observa-
tions that rapid correction may lead to central pontine myelinosis with paral-
ysis, coma, and death.
(d) Severe cases may need 3% (hypertonic) saline which contains 514 mmol/L
of sodium in aliquots of 3–5 mL/kg over 15 min–1 h to raise the plasma
sodium by 2–4 mmol/L and only to return the plasma concentration to
125 mmol/L.
(e) In less severe cases, management depends on fluid status. Hypovolemia
requires 0.9% saline. In normovolemic and hypervolemic states, correction
is done by fluid restriction with or without diuretics.

60
Q

HR 105, BP 155/89, Sats 94%.
Hb 9.9 g/dL, Hct 29.7%, Na 120 mEq/L, K 3.2 mEq/L, Ca 9.1 mg/dL, BUN
7 mg/dL, glucose 100 mg/dL.
Above values are obtained in a patient in PACU 30 min post-TURBT. The patient is extremely drowsy and not arousable. No medications were given in PACU
5. How would you manage this condition?

A

Management:
(a) Correct underlying cause.
(b) If severe, expert help should be obtained.
(c) The magnitude and rapidity of correction is controversial due to observations that rapid correction may lead to central pontine myelinosis with paralysis, coma, and death.
(d) Severe cases may need 3% (hypertonic) saline which contains 514 mmol/L
of sodium in aliquots of 3–5 mL/kg over 15 min–1 h to raise the plasma sodium by 2–4 mmol/L and only to return the plasma concentration to 125 mmol/L.
(e) In less severe cases, management depends on fluid status. Hypovolemia
requires 0.9% saline. In normovolemic and hypervolemic states, correction
is done by fluid restriction with or without diuretics.

61
Q

What is SIADH?

A

○ The syndrome of inappropriate ADH secretion (SIADH) is characterized by
hyponatremia, low plasma osmolality with normovolemia, and an inappropriately high urine osmolality.
○ It may be seen in the presence of malignant tumors which produce ADH-like substances, in neurological disorders (head injury, tumors), and in pneumonia. ○ Some drugs (barbiturates, opioids, chlorpropamide, anticonvulsants, and indomethacin) may also potentiate or increase ADH effect.
○ Treatment is with fluid restriction, and in severe or resistant cases, demeclocycline or lithium may be tried

62
Q

A 63-year-old man with hypertension, diabetes, and generalized weakness presents
for resection of small bowel. The patient’s medications include furosemide, metoprolol, and acetazolamide.
Lab values: HR 90, BP 105/65, Sats 96%, Hb 11 g/dL, Hct 31%, Na 130 mEq/L,
K 2.3 mEq/L, and Cr 2.0
His EKG shows the following rhythm.
1. What are your concerns regarding his EKG?

A

○ The clinical presentation along with EKG features (prominent U waves and apparent QT/U prolongation) suggests significant hypokalemia.
○ Hypokalemia is defined as plasma potassium of less than 3.5 mEq/L.
○ For every 0.3 mEq/L decrease in plasma potassium, the total body potassium
stores decrease by 100 mEq/L.
○ Mild hypokalemia is serum potassium >2.0 mEq/L; Severe hypokalemia is serum potassium <2.0 mEq/L.
○ The electrocardiographic changes include:
°Early—decrease in T wave.
°Later—ST depression and T inversion. PR interval prolongation. U waves appear in mid precordial leads.
°Severe—U and T fuse producing giant U waves and apparent prolongation of QT interval which is actually QU interval.

63
Q
  1. What are the causes for this abnormality?
A

Causes:
(a) Inadequate intake: diet and alcoholism.
(b) Excessive renal loss: mineralocorticoid excess, Cushing’s syndrome, diuretics, hydrochlorothiazide and furosemide therapy, carbonic anhydrase inhibitors, chronic metabolic alkalosis, renal tubular acidosis, and
ureterosigmoidostomy.
(c) Gastrointestinal losses: vomiting and diarrhea, which are commonly implicated as nutritional deficiency causes; nasogastric suctioning; and villous adenoma [2, 3].
(d) β-Adrenergic agonists, insulin, and alkalosis (respiratory and metabolic) shift potassium to the intracellular space.
(e) The most common renal cause of hypokalemia is diuretic therapy when loop diuretics and thiazides are co-prescribed. Loop diuretics block the sodium-potassium-chloride cotransporter in the thick ascending limb of the loop of Henle, while thiazides block the sodium-chloride cotransporter in the distal
convoluted tubule

64
Q
  1. How do you emergently correct low potassium preoperatively in this patient?
A

⊙Hypokalemia treatment consists of oral or intravenous replacement of potassium.
°Mild hypokalemia (>2.0 mEq/L): infuse potassium chloride up to 10 mEq/h iv
°Severe hypokalemia (<2.0 mEq/L, ECG changes, intense skeletal muscle weakness): infuse potassium chloride up to 40 mEq/h iv, with continuous ECG monitor.
○ Total KCL required is determined by calculating the K deficit.
K deficit (mEq/L) = (Goal K – Measured K)/serum Creatinine × 100
K deficit (mEq/L) = (4.0−2.3)/2 × 100
= 1.7/2 × 100 = 85 mEq/L
= 85 mEq/L

65
Q
  1. How do you monitor and manage perioperatively? Hypokalaemia
A

(a) Monitoring
• EKG
• Plasma potassium levels
• ABG
• Peripheral nerve stimulator
(b) Maintenance
• Avoid hyperventilation.
• Avoid hyperglycemia.
• Avoid epinephrine and other beta-2 agonist.
• Avoid diuretics unless supplemented with potassium chloride.
.

66
Q
  1. What are the anticipated problems and concerns anesthetizing this patient? Hypokalaemia
A
  1. (a) Severe hypokalemia may lead to arrhythmias, ventricular tachycardia, and
    ventricular fibrillation [5].
    (b) Hypokalemic patients may be sensitive to vasodilators or cardiac-depressant
    effects of volatile anesthetics.
    (c) Potential for prolonged response to non-depolarizing muscle relaxants.
    (d) Digoxin toxicity can occur with low potassium levels.
    (e) Insulin therapy can lower potassium levels.
    (f) While treating hypokalemia, concurrent hypomagnesemia should also be
    corrected
67
Q

A 55-year-old, 110 kg male with a history of alcoholic cirrhosis was admitted with
mental status changes and a decrease in urine output over the last 2 days.
Labs from this admission showed: creatinine 3.4 mg/dL (was 1.1 mg/dL, a month
ago), blood urea nitrogen (BUN) 70 mg/dL (18 mg/dL, a month ago), serum biliru-
bin 3 mg/dL, potassium 5.7 mg/dL, and sodium 125 mEq/L. The patient was diag-
nosed with acute renal failure.
1. What is the initial step in the evaluation of this patient?

A

○ A careful history and physical examination frequently identify events and/or disease processes that result in decreased tissue perfusion that can lead to prerenal disease (e.g., vomiting, diarrhea, bleeding, or sepsis) or post-ischemic acute tubular necrosis (ATN).
○ Clinical setting may help identify the underlying cause of AKI (e.g., hypotension, sepsis, aminoglycoside therapy, NSAIDS, or the administration of radiocontrast media).
○ Physical examination may suggest hypovolemia, such as unexplained tachy-
cardia, dry mucous membranes, decreased skin turgor, cool extremities, and
orthostatic hypotension.
○ Other physical exam findings may reveal signs of heart failure or cirrhosis presenting with edema, ascites, and other signs of specific organ dysfunction or may reveal abdominal compartment syndrome.
○ Examination should also include ultrasonography of the bladder to rule out obstructive etiology

68
Q
  1. What are the basic diagnostic tests that are used to distinguish prerenal disease from acute tubular necrosis (ATN)?
A

There are three basic diagnostic tests:
(a) Urinalysis with sediment examination:
• Normal or near normal (hyaline and/or fine granular casts) in prerenal disease.
• Muddy brown granular, epithelial cell casts, and free renal tubular epithelial cells in ATN.
• RBC/WBC casts could suggest glomerulonephritis.
• WBC casts with eosinophils could suggest interstitial nephritis.
(b) Fractional excretion of sodium (FENa), and to a lesser degree, the urine
sodium concentration. The fractional excretion of urea may be helpful in
patients being treated with diuretics as FENa is increased by diuretics due to the natriuresis.
(c) Response to fluid repletion: This is the gold standard for the distinction
between prerenal disease secondary to volume depletion and post-ischemic
or nephrotoxic ATN. Return of the serum creatinine to the previous baseline
within 24 to 72 h after volume repletion represents prerenal disease, whereas persistent AKI represents ATN.

69
Q

How is FENa estimated? How does it help in diagnosis?

A

FENa= UNa × SCr × 100/SNa × UCr
○ By definition, FENa is the ratio between the quantity of Na excreted in the urine
relative to the amount filtered at the glomerulus.
○ Measuring urine sodium concentration alone is not sufficient, as the sodium concentration in urine varies with water reabsorption.
○ It is necessary to plug in the serum and urinary creatinine into the calculation, in order to calculate the amount of fluid and sodium that is filtered through to glomerulus.
○ Prerenal AKI can be due to intrarenal vasoconstriction, systemic vasodilation,
and volume depletion.
○ These patients will try to compensate and retain sodium and usually have a FENa of less than 1%. If any of the above insults continue and become intense, the blood supply to the renal tubules is severely reduced leading to acute tubular necrosis.
○ Once the tubules are damaged, they lose their ability to reabsorb sodium, and the FENa will usually be greater than 2–3%.
○ FENa is often used in the setting of acute renal failure to help distinguish
between prerenal (decreased renal perfusion) and intrinsic renal (ATN due to
renal hypoperfusion) causes.
○ In general, a FENa of <1% suggests prerenal disease, between 1 and 2% is indeterminate, and >2% suggests ATN.
○ There are some exceptions to this, but overall, the specificity of this test is >80%.
○ There are limitations to FENa.
° The threshold used to distinguish prerenal and intrinsic renal disease may vary; there are other causes of low FENa and salt-wasting conditions (like diuresis) affect urinary sodium levels.

70
Q

What is contrast-induced nephropathy, its risk factors and measures to decrease risk?

A

○ Contrast-induced nephropathy (CIN) is either a relative increase in serum creatinine from baseline value by 25% or an absolute increase of 0.5 mg/dL within 48 to 72 h after contrast exposure not attributable to other causes and must persist for 2 to 5 days.
○ FENa may vary widely and in the minority of patients with oliguric CIN, the FENa may be low despite lack of clinical evidence of volume
depletion.
○ Risk factors include pre-existing renal dysfunction, diabetes with renal dysfunction, age >70 years, cardiorespiratory disease, hypotension or dehydration, and nephrotoxic medications (NSAIDs or aminoglycosides).
• Contrast agent volume, route of administration (intra-arterial), hyperosmolarity, and multiple doses in 72 h also add to the risk.
○ Measures to decrease risk of CIN include prehydration with saline, using
the lowest dose of low osmolar contrast, IV bicarbonate infusion, N-acetylcysteine (controversial), discontinuation of nephrotoxic drugs for 48h prior to contrast, and the use of hemofiltration (expensive) pre- and post-contrast use.

71
Q

What is FEUrea? When is it used?

A

○ Fractional excretion of other substances such as urea and uric acid can also be measured to determine their renal clearance to help distinguish prerenal from intrinsic renal causes.
○ The FEUrea may be more accurate in distinguishing ATN from prerenal disease in patients being treated with diuretics since diuretics as mentioned earlier cause natriuresis.
FEUrea percent=
Urinary urea × Serum Creatinine/
Serum urea Urinary Creatinine
× 100
○ FEUrea is 50 to 65% (>0.5) in acute tubular necrosis (ATN) and usually below 35% in prerenal disease [1, 2].

72
Q

A 55-year-old, 110 kg male with a history of alcoholic cirrhosis was admitted with mental status changes and a decrease in urine output over the last 2 days.
Labs from this admission showed: creatinine 3.4 mg/dL (was 1.1 mg/dL, a month ago), blood urea nitrogen (BUN) 70 mg/dL (18 mg/dL, a month ago), serum bilirubin 3 mg/dL potassium 5.7 mg/dL, and sodium 125 mEq/L. The patient was diag-
nosed with acute renal failure.
What are the causes of ARF in this patient?

A

○ The differential diagnosis of acute kidney injury (AKI) or acute renal failure (ARF) in this patient with cirrhosis includes prerenal azotemia, acute tubular necrosis, and hepatorenal syndrome (HRS). °Prerenal azotemia is caused by
hypovolemia (e.g., aggressive diuresis, diarrhea, and/or gastrointestinal bleeding) or by other causes of decreased effective blood volume induced by infections or vasodilators.
°Prerenal azotemia responds to volume expansion, and vasoconstrictors and dialysis are not required.
°Acute tubular necrosis mostly occurs in patients presenting with shock or a history of exposure to nephrotoxins/contrast agents. Acute tubular necrosis is treated with renal replacement therapy if indicated

73
Q

Is hepatorenal syndrome (HRS) a type of ATN?

A

○ Renal dysfunction in HRS is functional. ○ The pathophysiology of cirrhosis
involves portal hypertension leading to splanchnic arterial vasodilatation.
° The resultant primary systemic arterial vasodilatation leads to systemic hypotension which in turn causes activation of the neurohumoral axis with stimulation of the renin–angiotensin–aldosterone system (RAAS), sympathetic nervous system
(SNS), and arginine vasopressin (AVP).
° Stimulation of the RAAS, SNS, and
AVP contributes to maintenance of blood pressure by increasing systemic vascular resistance along with the secondary increase in cardiac output.
° While this compensatory neurohumoral activation attenuates any hypotension secondary to arterial vasodilatation, renal vasoconstriction with sodium and water retention also occurs.
° This resultant diminished renal function is, however, of a functional nature and thus should not be considered ATN in the initial phases.
° Prolonged and severe HRS can then lead to ATN

74
Q

What is RIFLE criteria and AKIN classification?

A

○ The RIFLE criteria were created by the Acute Dialysis Quality Initiative
(ADQI) in 2002 to define AKI.
○ The limitations include a need for a baseline creatinine level, smaller increases in creatinine (0.3 mg/dL) which can worsen outcome is not included in RIFLE; it is a retrospective tool and does not discriminate between the nature or the site of AKI.
○ In 2007 the Acute Kidney Injury Network changed the RIFLE criteria to stages in AKI.
• Stage one—Increase in serum creatinine of more than or equal to 0.3 mg/dL or increase to more than or equal to 150% to 200% (1.5- to 2-fold) from baseline.
Urine output <0.5 mL per kg per hour for more than 6 h.
• Stage two—Increase in serum creatinine to more than 200% to 300% (>2- to 3-fold) from baseline. Urine output <0.5 mL per kg per hour for more than 12h.
• Stage three—Increase in serum creatinine to more than 300% (>3-fold) from baseline (or serum creatinine of more than or equal to 4.0 mg/dL or commencement of acute renal replacement therapy (irrespective of the preceding increase in serum creatinine level or urine output).
○ More recently, the Kidney Disease: Improving Global Outcomes (KDIGO)
Acute Kidney Injury Work Group proposed changes to the staging for AKI. KDIGO covers both the AKIN and RIFLE criteria, taking into account changes in creatinine within 48 h or a decline in the glomerular filtration rate (GFR) over 7 days.
○ RIFLE, AKIN, and KDIGO scores were all good predictors of mortality in critically ill patients, and there were no differences among them in terms of predicting death.

75
Q
  1. How does substance abuse affect anesthesiology as a specialty?
A

○ Eighty percent of anesthesia residency programs have at least one resident with substance abuse.
○ One to two percent of anesthesia residents have a problem with substance abuse.
○ The Massachusetts General Hospital has instituted a preplacement (preemployment) and post-employment random urine testing in an attempt to lower the incidence of substance abuse among anesthesia residents.
○ Twenty-nine percent of anesthesia residents relapse after being allowed to return/continue in an anesthesia residency program.
○ For residents that are allowed to
return to their residency program, the initial presentation is death (10%).
○ Forty-three percent of program directors feel that residents in recovery should be allowed to attempt reentry, while 30% feel that they should not

76
Q

What test would you order for suspected fentanyl substance abuse in your patients?

A

○ Fentanyl can be detected by radioimmunoassay or more selective gas chromatographic techniques.
○ Urine and blood screening involving a novel enzyme-linked immunosorbent assay (ELISA) coupled with nanoparticles for fentanyl detection has the advantage of being simple, sensitive, inexpensive, and capable of detecting metabolites.
○ Fentanyl concentrations as low as 5 pg/well can be detected in urine and serum samples.

77
Q

What test would you order for suspected cocaine substance abuse in your patients?

A

○ Urine radioimmunoassay test is the initial screening test for cocaine abuse and is positive for up to 72 h after exposure
° Benzoylecgonine is the main cocaine urine metabolite tested in cocaine drug screening.
○ Gas chromatography coupled with mass spectrometry (GC-MS) or liquid chromatography coupled with a mass spectrometry (LC/MS) is more sensitive and sophisticated than immunoassay and can be done for confirmation of various drugs and their metabolites including cocaine.
° The window of testing is 1–3 days for urine testing.
○ Although hair testing is the most sensitive for cocaine and has the widest detection window indicating chronic use, it is not done routinely compared to urine.

78
Q

How do you test for marijuana drug use in your patients?

A

○ Urine is the preferred medium to test for marijuana use because of higher concentrations, longer detection time of metabolites, ease of sampling, and higher
sensitivity compared to blood .
○ The major metabolite tested in marijuana use is tetrahydrocannabinol (THC) and carboxy tetrahydrocannabinol (THCCOOH).
○ The detection window for urine testing is 10 h for THC and 25 days for THCCOOH.
○ While immunoassay is adequate for preliminary testing, advanced
chromatographic techniques are used for quantitation of levels.

79
Q

According to the American College of Surgeons certified Level I Trauma Centers,
what percentage of patients screened are positive for both alcohol and illicit drug
misuse?

A

Eleven percent of the patients screened at Level I Trauma Centers are found to be
positive for both legally intoxicated levels of alcohol and illicit drugs [6]. The use
of alcohol and illicit drugs is especially concerning due to higher incidence of
fatal and nonfatal motor vehicle accidents and higher perioperative morbidity.

80
Q

How do you group illicit drugs and what are their anesthetic implications?

A

○ Illicit drugs may be grouped into opioids, barbiturates, cocaine, benzodiazepines, ephedrine groups, cannabinoids, and hallucinogenic drugs
1. Opioid drugs (codeine, oxycodone, pentazocine, fentanyl, propoxyphene, methadone, heroin, morphine, meperidine) are used for analgesia.
° They can cause euphoria, respiratory depression, seizures, stupor, coma, and death.
° Treatment for opioid withdrawal includes clonidine, diphenhydramine, doxepin, and/or opioids (methadone or buprenorphine).
2. Barbiturates (secobarbital, pentobarbital, phenobarbital) are central nervous system depressants and can cause sedation, hypotension (central vasomotor depression and cardiac depression), and altered drug metabolism (fluoride, warfarin, digitalis, phenytoin).
3. Cocaine is a central nervous system stimulant (arterial vasoconstriction) with anesthetic concerns for increased MAC, sympathetic hyperactivity causing
hypertension/hypotension, tachycardia, increased myocardial oxygen demand, myocardial infarction, cardiac depression, angina, coronary spasms, thrombus,
arrhythmias, and death.
° Other anesthetic implications include psychosis, nasal septum perforation, restlessness, anxiety, irritability, confusion, pupillary dilatation, seizures, asthma, and pulmonary hemorrhage.
4. Benzodiazepines (diazepam, midazolam, flunitrazepam) are antianxiety agents and may result in respiratory depression especially with concurrent opioid use.
5. Ephedrine drugs (pseudoephedrine, methamphetamines) can cause hypertension, cardiac arrhythmias, dilated pupils, hyperthermia, and cardiac arrest in the perioperative setting.
° The response to treatment of hypotension with vasopressors is unpredictable in amphetamine-abusing patients.
° Acute intake of amphetamines increases the minimum alveolar concentration (MAC) of potent inhaled anesthetics.
° In contrast, chronic intake decreases the dose requirement for general anesthetic.
6. Cannabinoids have some antiemetic and analgesic properties and are the most commonly abused drugs.
° They can cause hallucinations and severe perioperative complications including cardiac arrhythmias, cardiac depression, hypotension, bradycardia, respiratory depression, bronchospasm, and pulmonary edema.
○ Hallucinogens [lysergic acid diethylamide (LSD), phencyclidine (PCP, ket-amine), psilocybin, mescaline, 3,4-methylenedioxymethamphetamine (MDMA, ectasy), γ-hydroxybutyrate (GHB)] are psychedelic drugs used for recreational purposes.
° They can cause anxiety, paranoia, delusions, panic attacks, and psy-chosis. ° The effects of acute drug intake usually develop over 1 to 2 h and last for
approximately 12 h.
° Ingestion of these drugs activates the sympathetic nervous system causing increased body temperature, tachycardia, hypertension, and dilated pupils.
° This is treated with fluids, pressors, vasodilators, and sympatholytics. ° Exaggerated response to sympathomimetic drugs should be expected.
° These drugs also prolong the analgesic and respiratory depressants effects of opioids.
° Inhibition of plasma cholinesterase activity can cause prolongation of
succinylcholine action in some patients.

81
Q

A 78-year-old male underwent an open AAA repair for an 8 cm infrarenal aneurysm.
His preoperative echo showed normal EF and no wall motion abnormality. During the surgery he had an episode of surgical bleeding and hypotension with transient ST depression which resolved with hypotension treatment and PRBC transfusion.
On postoperative day 1, the patient developed chest pain with ST depression and elevated cardiac troponin I (cTnI) which was 15 ng/mL. An echocardiography showed severe anteroseptal hypokinesia.
1. Has this patient suffered an acute myocardial infarction (MI)?

A
  1. Yes, this patient has elevated cardiac biomarkers along with symptoms of cardiac
    ischemia (chest pain) with imaging evidence of regional wall motion abnormality in the anterioseptal wall.
    ○ The term acute MI is used when there is evidence of myocardial necrosis in a clinical setting consistent with acute myocardial ischemia, detection of a rise and/or fall of cardiac biomarker values (preferably cardiac troponin) above the 99th percentile upper reference limit with at least one of the following:
    (a) Symptoms of cardiac ischemia
    (b) EKG changes: new significant ST-segment–T wave changes, new left bundle branch block, or development of pathological Q waves
    (c) Imaging evidence of new regional wall motion abnormality
    (d) Identification of an intracoronary thrombus by angiography or autopsy
82
Q

What are troponins?

A

○ Troponins are protein molecules that are part of cardiac and skeletal muscle.
○ Smooth muscle cells do not contain troponins.
○ Three types of troponins exist—
troponin I, troponin T, and troponin C. Each subunit has a unique function:
° Troponin T binds the troponin components to tropomyosin, troponin I inhibits the interaction of myosin with actin, and troponin C contains the binding sites for Ca2+ that help initiate contraction. Raised troponin levels indicate cardiac muscle cell injury and/or death as the molecule is released into the blood upon injury to the heart.
○ Troponins will begin to increase within 3 h following an MI.
○ The recommended cutoff value for an elevated cardiac troponin is the 99th percentile of a control reference group. As the troponin test kits are made by many manufacturers, the cutoff values suggested by the laboratory should be used as reference.

83
Q

What is the 2012 universal classification of myocardial infarction?

A

Universal Classification of Myocardial Infarction
○ Type 1 (spontaneous myocardial infarction): ° Spontaneous myocardial infarction related to atherosclerotic plaque rupture, ulceration, erosion, or dissection with resulting intraluminal thrombus in one or more of the coronary arteries leading to decreased myocardial blood flow or distal platelet emboli with ensuing myocyte necrosis.
° The patient may have underlying severe CAD but on occasion non-obstructive or no CAD.
○ Type 2 (myocardial infarction secondary to an ischemic imbalance):
° In instances of myocardial injury with necrosis where a condition other than CAD contributes to an imbalance between myocardial oxygen supply and/or demand, e.g., coronary endothelial dysfunction, coronary artery spasm, coronary embolism, tachy-/brady-arrhythmias, anemia, respiratory failure, hypotension, and
hypertension with or without LVH.
○ Type 3: Myocardial infarction results in death when biomarker values are unavailable.
○ Type 4a: Myocardial infarction related to percutaneous coronary intervention (PCI).
○ Type 4b: Myocardial infarction related to stent thrombosis.
○ Type 5: Myocardial infarction related to coronary artery bypass grafting (CABG)

84
Q
  1. What conditions can raise troponin levels other than MI?
A
  1. Causes of troponin elevation other than MI include the following:
    (a) Myocarditis
    (b) Pericarditis
    (c) Cardiac contusion/trauma
    (d) Aortic dissection
    (e) Endocarditis
    (f) Cardiac surgery
    (g) Pulmonary embolism
    (h) Stroke (ischemic or hemorrhagic)
    (i) Cardiopulmonary resuscitation (CPR)
    (j) Defibrillation
    (k) Chronic severe heart failure
    (l) Cardiac arrhythmias (tachyarrhythmias, brady-arrhythmias, heart blocks)
    (m) Sepsis
    (n) Renal failure
    (o) Hypertrophic obstructive cardiomyopathy (HOCM)
    (p) Takotsubo cardiomyopathy
    (q) Burns
    (r) Extreme exertion
    (s) Infiltrative diseases such as amyloidosis
    (t) Medications and toxins such as doxorubicin, trastuzumab, and snake venom
    (u) Transplant vasculopathy
    (v) Critical illness
85
Q

A 78-year-old male underwent an open AAA repair for an 8cm infrarenal aneurysm. His preoperative echo showed normal EF and no wall motion abnormality. During the surgery he had an episode of surgical bleeding and hypotension with transient ST depression which resolved with hypotension treatment and PRBC transfusion. On postoperative day 1, the patient developed chest pain with ST depression and elevated cardiac troponin I (cTnI) which was 15ng/mL.An echocardiography showed severe anteroseptal hypokinesia.
How would you classify this patient’s perioperative MI?

A
  1. This patient has suffered a perioperative MI, either Type 1 or type 2.
    Acute Coronary Syndrome (Type 1 PMI)
    ○ Acute coronary syndrome occurs when an unstable or vulnerable plaque undergoes spontaneous rupture, fissuring, or erosion, leading to acute coronary
    thrombosis, ischemia, and infarction.
    ○ Although it is currently widely accepted
    that intraplaque inflammation plays a pivotal role in plaque instability and spontaneous acute coronary syndrome, external stressors such as those occurring postoperatively are believed to contribute.
    (a) Physiological and emotional stresses are known to predispose to MI, likely because of the sympathetic induced hemodynamic, coronary vasoconstrictive, and prothrombotic forces thought to promote plaque disruption.
    ○These conditions are common perioperatively.
    ⊙ Catecholamines and cortisol increase after surgery and may remain elevated for days. Stress hormones increase with pain, surgical trauma, anemia, and hypothermia.
    (b) Tachycardia and hypertension, common in the perioperative period,
    may exert shear stress, leading to rupture of plaques with outward (positive) remodeling, thin fibrous caps, and high circumferential tensile stress or to endothelial stripping/erosion caused by high blood velocities around plaques with inward (negative) remodeling and
    severe coronary stenosis.
    ○ Myocardial Oxygen Supply-Demand Imbalance (Type 2 PMI)
    °Numerous studies using perioperative °Holter monitoring in high-cardiac-risk
    patients undergoing major surgery showed that silent, heart rate-related ST-
    segment depression is common postoperatively and is associated with in-hospital and long-term morbidity and mortality.
    ° Postoperative cardiac complications, including sudden death, occurred after prolonged silent ST-segment depression.
    ○ These findings were further corroborated by studies that correlated continuous, online 12-lead ST-segment analysis with serial cardiac troponin measurements after major vascular surgery.
    °Cardiac troponin elevations occurred after prolonged transient, postoperative ST-segment depression, and peak troponin elevations correlated with the duration of ST depression.
    ° ST elevation occurred in <2% of postoperative ischemic events and was a rare cause of PMI. Hence, prolonged, ST-depression-type ischemia is the most common cause of PMI.
86
Q

What should be done next? Mx of perioperative MI

A

There are no clear algorithms or guidelines to navigate treatment in perioperative MIs. This is also addressed in Chap. 15.
○ The cardiology team is consulted and involved early.
○ Steps should be taken to ensure adequate oxygen supply to the myocardium by addressing issues with oxygen saturation and anemia.
° Myocardial oxygen demand should be reduced by controlling tachycardia and hypertension.
○ There are no randomized controlled trials to clarify if PCI or systemic anticoagulation is beneficial in the perioperative setting.
° Retrospective analysis of over 1000 cases of perioperative MI showed that the 30-day mortality of patients who received diagnostic catheterization was 5.2%. Those who received PCI had a 30-day mortality of 11.3%. Part of this mortality is attributed to the large postopoperative hemorrhage risk of anticoagulation.
○ Clinical decision making should be made on a case by case risk benefit analysis.

87
Q

A 67-year-old man presented to the ER with increasing shortness of breath, tired-
ness, and weight gain. Immunoassay for BNP showed a value of 800 pg/mL.
What is BNP and NT-proBNP?

A

○ B-type natriuretic peptide is also called brain-type natriuretic peptide (BNP) as
it was first described in 1988 after isolation from porcine brain.
° However, it was soon found to originate mainly from the heart.
○ B-type natriuretic peptide (BNP) and N-terminal proBNP (NT-proBNP) are released by the ventricular myocardium in response to myocardial wall stress initially as a 108 amino acid prohormone.
° It is cleaved by enzymes corin/furin to BNP the 32 amino-acid, biologically
active part of the prohormone and NT-proBNP which is the 76 amino acid, biologically inactive compound.
○ BNP produces a variety of biological effects by interaction with the natriuretic peptide receptor type A (NPR-A) causing intracellular cGMP production.
° These include natriuresis/diuresis, peripheral vasodilatation, and inhibition of the renin–angiotensin–aldosterone system (RAAS) and the sympathetic nervous system (SNS).
° All effects ultimately lead to decreased afterload.
○ BNP has a half-life of 20 min and is cleared by binding to the natriuretic pep-
tide receptor type C (NPR-C) and through proteolysis by endopeptidases.
○ NT-proBNP has a half-life of 120 min and is cleared by renal excretion.

88
Q

What are the normal levels and conditions that cause elevated levels? BNP & NT

A

○ BNP levels are normally less than 100 pg/mL and the NT-proBNP is less than
300 pg/mL.
○ The levels are higher in:
(a) females due to differences in metabolism
(b) advancing age
(c) worsening renal function (NT-proBNP affected more due to renal clearance)
(d) LV hypertrophy
(e) Systolic and diastolic dysfunction
(f) Fluid overload BNP and NT pro BNP serve as good markers of heart failure.
○ The levels for both markers are different to exclude or confirm the diagnosis of heart failure.
○ BNP—level < 100 pg/mL heart failure (HF) unlikely; level > 500 pg/mL HF very likely.
° Levels 100–500 use clinical judgment.
° NT-proBNP—level < 300 pg/mL HF unlikely
° Age < 50 years, level > 450 pg/mL—HF likely
° Age 50–75 years, level > 900 pg/mL—HF likely
° Age > 75 years, level > 1800 pg/mL—HF likely
A good correlation has been made between increasing levels of BNP and
functional class of NYHA classification as depicted in the Fig. 35.1.

89
Q

How do these markers aid in the diagnosis of heart failure?

A

○ BNP and NT pro BNP serve as good markers of heart failure. The levels for both markers are different to exclude or confirm the diagnosis of heart failure.
○ BNP—level < 100 pg/mL heart failure (HF) unlikely; level > 500 pg/mL HF
very likely.
° Levels 100–500 use clinical judgment.
° NT-proBNP—level < 300 pg/mL HF unlikely
° Age < 50 years, level > 450 pg/mL—HF ° likely
° Age 50–75 years, level > 900 pg/mL—HF likely
° Age > 75 years, level > 1800 pg/mL—HF likely
° A good correlation has been made between increasing levels of BNP and
functional class of NYHA classification as depicted in the Fig. 35.1.

90
Q

What are the other uses outside diagnosing heart failure? BNP NT-BNP

A

○ BNP and NT-proBNP provide strong prognostic information, and elevated levels are associated with an unfavorable outcome (death, sudden cardiac death, readmission, or cardiac events) in patients with heart failure or asymptomatic left ventricular dysfunction.
○ They are also useful for choosing optimal treatment and monitoring its effects in heart failure.

91
Q
  1. What is heart failure and why is it important to diagnose it?
A
  1. ○ Heart failure affects approximately 5.7 million Americans, and about 670,000
    new cases are diagnosed annually in the United States.
    ○ It is a leading cause of hospital admissions and readmissions in people over 65 years.
    ○ The estimated total health-care cost of HF in the United States in 2010 was $39.2 billion or 1–2% of all health-care expenditures.
    ○ The risk of death is about 35% in the year after diagnosis after which it decreases to below 10% each year.
    ○ HF is either diastolic, decreased left ventricular filling, or systolic, decreased
    pump function.
    ○ It is a diagnosis that is made clinically by history (breathlessness and fatigue) and physical exam (elevated jugular venous pressure and lung crackles).
    ○ These features are not sensitive or specific, and there is no gold standard investigation to make the diagnosis.
    ○ The severity is classified based on
    symptoms and functional limitations into four grades according to the New York
    Heart Association.
    ○ Patients with heart failure suffer a decreased quality of life with significantly reduced physical and mental health.
    ○ So, early diagnosis of heart failure, identification of the cause to determine reversibility, and institution of appropriate management strategies which include lifestyle changes, medications, and/or surgery can potentially make a big impact on the quality and duration of life.
92
Q

Are there limitations? BNP

A

Limitations include false low levels in:
(a) Obesity
(b) Early acute heart failure
(c) Heart failure due to causes upstream from the left ventricle, e.g., mitral valve or pericardial disease
False high levels as already mentioned in:
(a) Females
(b) Advancing age
(c) Renal failure

93
Q
  1. What does the data in Fig. 30.1 show?
A

The data shown represents a typical thromboelastography (TEG) trace.
Thromboelastography is a viscoelastic hemostatic assay that measures global
properties of whole blood clot formation. It shows the interaction of platelets
with the coagulation cascade including aggregation, clot strengthening, fibrin
cross-linking, and fibrinolysis. TEG is an effective and convenient means of
monitoring whole blood coagulation and provides a global assessment of hemo-
static function.
It can assist in determining if a patient has normal hemostasis or is bleeding
due to a coagulopathy or anticoagulant therapy.

94
Q
  1. When can it be used?
A

Conventional coagulation tests like PT and aPTT poorly reflect in vivo hemosta-
sis. As they are performed in plasma, they assess only a portion of the coagula-
tion system and do not provide information on the full balance between
coagulation and anticoagulation. In contrast TEG is performed on whole blood
and provides information on clot formation, stabilization, and dissolution thus
assessing coagulation and fibrinolysis.
TEG is used to assess hemostasis during liver transplantation, postpartum hem-
orrhage, cardiac surgery, and in trauma resuscitation. It can also be used in coagu-
lopathy due to other reasons such as sepsis and guide management. TEG can also
provide information on the presence and adequacy of platelet inhibition.
In cardiac surgery during cardiopulmonary bypass, abnormal coagulation can
be identified before heparin reversal with the addition of heparinase to the testing.
This will be useful in long pump runs, in deep hypothermia, in the presence of
ventricular assist devices, and in major vascular procedures. In pinpointing the
specific problem, TEG has been shown to reduce blood transfusion in cardiac
surgery [1, 2]

95
Q
  1. How is it produced?
A

The test sample is placed in an oscillating cup (4–45° every 5 s) heated to
37 °C. A pin is suspended into the sample by a torsion wire which is attached to
a mechanical/electrical transducer. The elasticity and strength of the developing
clot changes the motion of the pin which is converted into a graphical and
numerical output which is displayed.

96
Q
  1. What do the parameters R, K, α, MA, and A30 (LY30) indicate?
A

The above tracing can be looked at in the following phases.
(a) Initial clot formation
Split point (SP) time = from the start of the test to the split of the trace
(first detectable fibrin)
Reaction (R) time = from the start till the trace reaching 2 mm amplitude
(1 mm either side of baseline) and represents continued production of throm-
bin and conversion of fibrinogen to fibrin (normal 4–9 min kaolin activated).
Prolonged R in factor deficiency and anticoagulants.
R-SP = delta (Δ). Thrombin burst. Low delta indicates hypercoagulability
and vice versa (normal 0.7–1.1 min). The test can be performed with hepari-
nase (TEG-H), and a difference of more than 2 min between R value of TEG
and TEG-H indicates heparin effect.(b) Conversion of fibrinogen to fibrin
Kinetics (K) time = time interval between 2 mm and 20 mm on the
trace and represents fibrin cross-linkage and rate of bonding between
fibrin and platelets and is a measure of fibrinogen function (normal
1–3 min kaolin activated). Prolonged by anticoagulants, hypofibrinogenemia and thrombocytopenia and shortened by increased fibrinogen level
and platelet function.
Angle (α) = the angle at which the curve rises from SP to K and is related
directly to K time as a measure of fibrin platelet interaction and therefore
functional fibrinogen (normal 59–74°)
(c) Clot strength
Maximum amplitude in mm (MA) = represents clot strength and platelet
function (normal 55–74 mm).
G = this is calculated from platelet performance (MA) and expressed as
resistance unit (normal 5.3–13.2 dynes/cm2).
(d) Clot lysis
A30, LY30, EPL30 = percentage amplitude at 30 min post-MA (A30). Clot
lysis begins a short period after MA is reached and continues for about
15 min. The software estimates the percent lysis during this period (EPL30).
After 30 min the EPL30 becomes the percent lysis LY30 (normal 0–7.5%).

97
Q
  1. How can the above parameters R, K, α, MA, and A30 (LY30) be used to guide therapy?
A
  1. Increased R time—FFP (one unit of FFP will decrease R time by 2.5 min)
    Decreased angle—cryoprecipitate
    Decreased MA—DDAVP for slight decrease and platelets for more signifi-
    cant decrease (one unit of platelets increase MA by 7–9 m)
    Fibrinolysis—tranexamic acid, aminocaproic acid, or aprotinin
98
Q
  1. What is platelet mapping and how does it help?
A

○ Platelet mapping provides the degree of inhibition of platelets via the ADP (Plavix) and AA (Aspirin) pathways and thereby the effectiveness of the antiplatelet drugs which cannot be done with routinely available coagulation tests.
○ This involves four separate (channels) analyses.
(a) Baseline—kaolin activates platelets maximally as in regular TEG.
(b) Activator—heparin (via heparinized tube to draw the sample) to inhibit thrombin and activator to convert fibrinogen to fibrin.
• Activator is reptilase and Factor XIIIa (contribution of fibrin to clot).
(c) ADP—like previous (for fibrin clot) plus ADP added to activate platelets via the GP IIb/IIIa receptor.
(d) AA—activator for fibrin clot and arachidonic acid to activate thromboxane
A2 pathway producing platelet aggregation.
• It is possible to determine the (percent) platelet inhibition by comparing the rela-
tive strengths of the clots in the activator, ADP or AA cups with the baseline.
• The residual function (percent aggregation) is multiplied by the baseline G to get the net G or net clot strength.
• The goal is to maintain it between 5 and 9 to prevent bleeding or thrombosis.

99
Q

How does thromboelastography (TEG®) differ from rotational thromboelastom-
etry (ROTEM®)?

A
  1. While TEG and ROTEM both measure the viscoelastic hemostasis of whole
    blood ROTEM is a modern modification of TEG. There are differences in the
    operations of the testing and the nomenclature. In ROTEM, the heated cup with
    the sample is fixed, but the pin suspended on a ball bearing mechanism rotates
    through an arc of 4–75° every 6 s, and the clot characteristics are measured by an
    optical sensor.
    ROTEM can measure four samples, while TEG can measure two samples
    simultaneously.
    ROTEM utilizes automated pipetting, while TEG requires manual pipetting.
    TEG is sensitive to vibrations, whereas ROTEM is resistant to mechanical shocks.
    Nomenclature (not interchangeable) for ROTEM as compared to TEG:
    Clotting time (CT) = R (reaction) time (time for trace to reach 2 mm in both)
    Clot formation time (CFT) = K (kinetics) time (time for trace to reach 20 mm in
    both)
    α angle = α angle
    Maximum clot firmness (MCF) = maximum amplitude (MA)
    Lysis = lysis index 30 (LI30) is the percent reduction in MCF 30 min after CT in
    ROTEM, whereas in TEG LY30 and LY60 (lysis 30 and lysis 60) are the percent
    reductions of the curve 30 and 60 min after MA is reached.
    Additional ROTEM assays:
    INTEM = contact activation, information similar to APTT
    EXTEM = tissue factor activation, information similar to PT
    HEPTEM = assesses heparin effect
    APTEM = in conjunction with EXTEM assesses fibrinolysis
    FIBTEM = in conjunction EXTEM allows qualitative analysis of the contribu-
    tion of fibrinogen to clot strength independent of platelets [4]
100
Q
A

The low MA and G demonstrate hypocoagulable platelet function. If the patient
is bleeding, platelets and/or DDAVP may be indicated.

101
Q
A

This flatline tracing demonstrates a complete inability to form a clot and can be
a result of several things. Clinically, it can be an extreme deficiency/suppres-
sion of factor function due to hemorrhage or over-anticoagulation. It could also
be a technical error in the running of the test (not adding calcium to a citrated
sample, or not loading the cup/pin properly). If it is due to any of the clinical
reasons mentioned, FFP or reversal of anticoagulation would be indicated.

102
Q
A

This tracing shows a minor decrease in factor function (TEGACT) and a significant
reduction in platelet function with the low MA and G values. Fibrinogen function
is also affected slightly (angle and K). If the patient is bleeding, a dose of platelets
should not only improves platelet function but contains a unit of thawed plasma and
about 400 mg of fibrinogen, which should normalize the TEGACT and K values.
DDAAVP may be useful to get the maximum impact from the platelets.

103
Q
A
  1. This tracing demonstrates primary fibrinolysis.
    ○ The key criteria are a decreased
    MA and G and elevated fibrinolysis (LY30).
    ○ Treatment is an antifibrinolytic like Amicar or tranexamic Acid.
    ○ Factors demonstrate slightly hypercoagulability with the shortened R value, but no assessment of fibrinogen or platelet function can be made, until the fibrinolysis is corrected.
    ○ The antifibrinolytic will typically correct the lysis quickly.
104
Q
A
  1. This tracing shows prolonged TEGACT indicating either factor deficiency or
    anticoagulant effect.
    ○ Since this is a post-protamine sample, it may be due to residual heparin still circulating.
    ○ If the heparinase tracing shows a lower
    TEGACT value, then additional protamine would be indicated.
    ○ If the TEGACT does not shorten in the heparinase cup, then FFP would be indicated.
105
Q
A

This tracing shows hypercoagulability of factors (shortened R); fibrinogen (K
and angle), and platelets indicate slight hypercoagulability. Fibrinolysis (LY30)is also elevated. This could be indicating stage 1 DIC, or a patient with some
form of thrombus that is initiating the lytic system. The assessment of the
patient should focus on identifying the underlying cause of the hypercoagula-
bility and treating the cause, and possibly treating the hypercoagulability with
some form of anticoagulant.

106
Q
A

This tracing shows hypercoagulability of factors (R), fibrinogen (K and angle),
and platelets (MA and G). First priority is to identify the cause of the hyperco-
agulability. If it is thrombin driven (due to trauma, cancer, or some other inflam-
matory process), then anticoagulation may be the treatment of choice. If it is
likely platelet driven (stents or artificial surfaces exposed to the blood), then
antiplatelet agents may be the treatment of choice.

107
Q
A
108
Q
A

○ The citrated kaolin tracing (white) shows decreased function of all parameters.
○ This dysfunction significantly improves in the heparinase cup, leaving only milder factor deficiency (R), and slightly hypocoagulable fibrinogen function (K and angle).
○ Treatment would be protamine to treat the heparin effect, and possibly some FFP (depending upon presence or degree of bleeding).
○ The approximately 300 mg of fibrinogen contained within a bag of FFP will likely correct the fibrinogen values

109
Q
A

○ This set of tracings is an ADP study of the PlateletMapping® assay.
○ There is a slight prolongation of the R value (8.3; range: 2–8), but with 42.1% inhibition
on the ADP assay, the platelet function is not at the high end of normal, as the basic TEG would suggest.
○ Looking at the MA value of the ADP tracing (46.2 in the popup box), this is a little below the normal range for MA of 51–69 mm.
○ This would indicate decreased platelet function and, if the patient was going for
a procedure, an increased risk of bleeding with the procedure.
⊙ Options would be to delay procedure, if elective, until the inhibition decreases (if due to meds) or, if urgent, to proceed with platelets and/or DDAVP being available.

110
Q

A 36-year-old gentleman is admitted to the intensive care unit following major trauma secondary to a road traffic accident. In the course of resuscitation and emergency surgery, he required a 20-unit blood transfusion. Six hours following admission, it is noted that he is bleeding from wound and line puncture sites. The following lab blood tests results are received:
What is the differential diagnosis of a low platelet count?

A

○ Thrombocytopenia is defined as a platelet count of less than 150 × 109/L7.
○ It may be due to:
• A decreased production of platelets:
– Selective impairment of platelet production: drugs (alcohol, thiazide diuretics, cytotoxic drugs) and viral infections
– Generalized disease of the bone marrow: aplastic anemia or marrow infiltration in leukemia or dissemination cancer
• Decreased platelet survival
– With an immune basis: idiopathic thrombocytopenia purpura, systemic lupus erythematosus, drugs (heparin), and infections (infectious mononucleosis, HIV, CMV)
– Without an immune basis: disseminated intravascular coagulation, thrombotic thrombocytopenic purpura, and cardiopulmonary bypass
• Sequestration
– Hypersplenism
• Dilutional
– Following massive transfusion of stored blood

111
Q

A 36-year-old gentleman is admitted to the intensive care unit following major
trauma secondary to a road traffic accident. In the course of resuscitation and emer-
gency surgery, he required a 20-unit blood transfusion. Six hours following admis-
sion, it is noted that he is bleeding from wound and line puncture sites. The following
lab blood tests results are received:What is the likely cause of the low platelets in this patient, and how is this condi-
tion diagnosed?

A

○ The likely diagnosis in this patient is disseminated intravascular coagulation
(DIC).
○ The diagnosis of DIC is not made by the examination of a single laboratory marker but based on the combination of clinical history and a number of test results.
○ Features suggestive of DIC in the clinical history include the presence of clinical conditions known to trigger DIC (see below) and also the clinical presentation due to the resultant consumptive coagulopathy: widespread petechiae and ecchymosis and blood oozing from wound sites, intravenous lines, catheters, and surgical drains.
○ When injury to the pulmonary vasculature occurs, hemoptysis and acute respiratory distress syndrome may result.
○ Other serious complications of DIC include acute renal failure, thrombosis, gangrene, and loss of digits, intracerebral hematoma, and cardiac tamponade.
○ Laboratory features suggestive of DIC include:
• Rapidly declining platelet count
• Prolonged prothrombin (PT) time
• Prolonged activated partial thromboplastin (aPTT) time
• Low fibrinogen levels, although only a clinical feature of approximately 30% of the more severe cases
• Raised fibrin degradation products (FDPs) and elevated D dimer level

112
Q

A 36-year-old gentleman is admitted to the intensive care unit following major trauma secondary to a road traffic accident. In the course of resuscitation and emergency surgery, he required a 20-unit blood transfusion. Six hours following admission, it is noted that he is bleeding from wound and line puncture sites. The following lab blood tests results are received:
Which other clinical states are associated with it?

A

○ Disseminated intravascular coagulation is an acquired complication of an underlying illness where systematic activation of the coagulation system occurs when blood is exposed to procoagulants .
○ The process can be classified as either:
• Acute or decompensated DIC, which occurs over a short-time period
• Chronic or compensated DIC, where small amounts of procoagulant are released over longer-time period
○ Conditions commonly associated with DIC include:
• Sepsis and severe infection
• Trauma: severe tissue injury, head injury, fat embolism
• Cancer: myeloproliferative diseases and solid tumors, e.g. pancreatic or prostate carcinomas
• Obstetric complications: amniotic fluid embolism and placental abruption
• Vascular disorders: aortic aneurysm and giant hemangiomas
• Reaction to toxins: snake venom and drugs
• Immunologic disorders: severe allergic reaction, hemolytic transfusion reaction, and transplant rejection

113
Q

A 36-year-old gentleman is admitted to the intensive care unit following major
trauma secondary to a road traffic accident. In the course of resuscitation and emer-
gency surgery, he required a 20-unit blood transfusion. Six hours following admis-
sion, it is noted that he is bleeding from wound and line puncture sites. The following
lab blood tests results are received:Can you describe the pathophysiology behind this condition?

A

○ Disseminated intravascular coagulation is characterized by systemic activation of coagulation through the release of tissue thromboplastin or thromboplastic substances into the circulation.
○This leads to the widespread intravascular deposition of fibrin throughout the microcirculation.
○ As a result of this widespread thrombosis, there is a depletion of coagulation factors and platelets.
○ There is also a secondary pathological activation of fibrinolysis.
•Thus DIC may result in microinfarcts and tissue hypoxia caused by microemboli as well as a coagulopathy due to the deletion of factors required for hemostasis (consumption coagulopathy).

114
Q

A 36-year-old gentleman is admitted to the intensive care unit following major trauma secondary to a road traffic accident. In the course o resuscitation and emergency surgery, he required a 20-unit blood transfusion. Six hours following admission, it is noted that he is bleeding from wound and line puncture sites. The following lab blood tests results are received:
What are the principles of management of this condition?

A

○ Management of DIC should involve:
1. Treatment of the underlying cause
2. Supportive therapy and replacement of blood components [4]
○ The major focus of management of DIC is specific and vigorous treatment of the underlying disorder.
○ This may be an aggressive management of sepsis or an infective source, evacuation of the uterus in the case of intrauterine death or debridement of tissues in the case of severe burns or trauma.
○ Without management of the cause, treatment of DIC is likely to fail.
○ The decision to transfuse blood products should not be based on the results of coagulation tests alone but on the need to treat an actively bleeding patient.
○ Blood products may also be used as prophylaxis to prevent bleeding but the literature to support this use is limited.
○ Commonly transfused products to correct coagulopathy:
• Fresh frozen plasma: a standard dose of 10–15 mL/kg should be used during Active hemorrhage, aiming for an INR of less than 1.5 and an aPTT ratio of less than 1.5.
• Platelets: the current British Society of Haematology guidelines suggest that platelet counts should be maintained at over 75 × 10−9/L in patients who are bleeding. The platelet count is expected to rise by 30–50 × 10−9/L after the
transfusion of a single pooled unit.
• Cryoprecipitate: cryoprecipitate should be used in a bleeding patient when the fibrinogen level is less than 1.5 g/L. A standard transfusion of two bags of cryoprecipitate is expected to increase the fibrinogen concentration by 0.5 to 1 g/L.
○ The use of anticoagulants, such as heparin, has been proposed to interrupt the systemic activation of coagulation seen in DIC. However, clinical trials have so far failed to show benefit

115
Q

A 36-year-old gentleman is admitted to the intensive care unit following major trauma secondary to a road traffic accident. In the course of resuscitation and emergency surgery, he required a 20-unit blood transfusion. Six hours following admission, it is noted that he is bleeding from wound and line puncture sites. The following lab blood tests results are received:Most patients suffering this complication require extensive transfusion of blood
products.
What are the short- and long-term complications of transfusions of blood products?

A

○ The transfusion of blood products to support the hemoglobin level and correct coagulation abnormalities can be a life-saving intervention in DIC.
○ However, prior to any transfusion, full consideration should be given to its necessity; transfusion of blood products has been proven to have multiple long- and short-term complications.
○ Early complications of blood product transfusion
• Hemolytic reactions
– Immediate
– Delayed
• Acute nonhemolytic transfusion reactions
– Febrile
– Allergic
– Hypotensive
• Transfusion-related acute lung injury (TRALI)
• Transfusion-associated circulatory overload (TACO)
• Reactions secondary to bacterial contamination
Complications specific to massive transfusion (complete replacement of the
circulating blood volume in 24 h)
• Hypothermia
• Dilutional coagulopathy if inadequate coagulation factors replaced relative to packed red cells
• Hyperkalemia
• Citrate toxicity resulting in hypocalcemia and hypomagnesemia
Late complications of blood product transfusion
• Transmission of infection
– Viral (hepatitis A, B, and C, HIV and CMV)
– Bacterial (Treponema pallidum, Salmonella)
– Parasites (malaria, toxoplasma)
• Transfusion-associated graft-versus-host disease
• Immune modulation possibly resulting in worse outcomes in cancer recur-
rence and metastases
• Immune sensitization (Rhesus D antigen)

116
Q

A 36-year-old gentleman is admitted to the intensive care unit following major trauma secondary to a road traffic accident. In the course of resuscitation and emergency surgery, he required a 20-unit blood transfusion. Six hours following admission, it is noted that he is bleeding from wound and line puncture sites. The following lab blood tests results are received:
7. What is the prothrombin time, and how is it related to the INR?

A

○ The prothrombin time is an assay to evaluate factors within the extrinsic pathway of the coagulation cascade: II, V, VII, and X .
○ Tissue thromboplastin (a brain extract) and calcium are added to citrated plasma.
○ Clotting normally takes place in 10 to 12 s.
○ The prothrombin time has significant interlaboratory variability influenced by the thromboplastin used.
○ In an effort to offset variation in thromboplastin reagent and enhance standardization of PT in patients receiving oral anticoagulants, the World Health Organization (WHO) introduced the international normalized ratio (INR) in 1983.
○ The INR is a mathematical conversion of a patient’s PT that accounts for the sensitivity of the reagent used in a given laboratory, by factoring in the international sensitivity index (ISI).
○ Each manufacturer assigns an ISI value for any tissue factor they manufacture. The ISI value indicates how a particular batch of tissue factor compares to an international reference tissue factor
○ The ISI is usually between 0.94 and 1.4.
- The INR is then calculated using the following formula:
INR = [Patient PT/Mean PT]ISI
○ In this formula, patient PT is measured prothrombin time, mean PT is geometric mean PT of at least 20 healthy subjects of both sexes tested at a particular laboratory, and ISI is international sensitivity index that is specific to each reagent-instrument combination.

117
Q

A 75-year-old man is scheduled to undergo urgent laparotomy for small bowel obstruction. On preoperative examination he looks jaundiced, and you request a full liver function test profile including total bilirubin, AST, ALT, Alk Phos, gamma-glutamyl transferase (γGT), and glucose. The results show:
1. What are liver function tests, and what are the different ways we can monitor liver function via blood tests?

A

Liver function tests can [1]:
• Detect the presence of liver disease and dysfunction
• Distinguish between different types of liver disorders
• Monitor the extent of liver damage
• Monitor responses to treatment
Liver function tests can look at the liver’s function in the following ways:
• Hepatic cell death: transaminases
• Biliary tree: alkaline phosphatase and γGT
• Bilirubin manufacture, conjugation, and bile obstruction
• Synthetic function: albumin, coagulation, and glucose

118
Q

A 75-year-old man is scheduled to undergo urgent laparotomy for small bowel
obstruction. On preoperative examination he looks jaundiced, and you request a full
liver function test profile including total bilirubin, AST, ALT, Alk Phos, gamma-
glutamyl transferase (γGT), and glucose. The results show:
Why might the bilirubin be elevated?

A

○ Total bilirubin is made up of unconjugated and conjugated (often called direct) bilirubin.
• Unconjugated bilirubin is initially formed from heme, mostly from red cell
hemoglobin, and is hydrophobic so it is mainly albumin bound.
• It can also be made from muscle myoglobin, mitochondrial cytochromes, catalase, peroxidase, and nitric oxide synthase.
• The liver clears the blood of unconjugated bilirubin via hepatocyte conjugation to make it conjugated water-soluble bilirubin.
• This is secreted into the bile and subsequently the intestine where metabolism of conjugated bilirubin into urobilinogen and its reabsorption accounts for the yellow color of urine.
• The metabolism of urobilinogen into stercobilin while in the bowels accounts for the brown color of stool; hence having white or clay-colored stool may indicate a blockage in bilirubin processing and thus potential liver dysfunction or obstruction to bile (cholestasis).
○ Raised bilirubin above about 3 mg/dL causes jaundice (from the French “jaune,” yellow), the dark yellow pigmentation of the skin, sclerae, and
other mucous membranes resulting from excess bilirubin in the extracellular fluid.
○ Raised unconjugated bilirubin is caused by pathology prior to the conjugation process: hemolysis, abnormal erythropoiesis, reduced delivery of bilirubin to the liver (cardiac failure, drugs), and defective bilirubin conjugation (congenital syndromes and hyperthyroidism).
○ Conjugated hyperbilirubinemia occurs in individuals with hepatocellular
damage, biliary obstruction (either intra or extra hepatic), and sepsis.

119
Q
  1. What are AST and ALT?
A

○ The enzymes aspartate transaminase (AST) (also known as serum glutamic oxaloacetic transaminase, or SGOT) and alanine transaminase (ALT) (formerly called serum glutamic pyruvic transaminase or SGPT) are associated with liver parenchymal cells, and if the liver is damaged, the increased permeability of the hepatocyte membrane causes enzyme leakage out into the systemic circulation.
○ ALT is mainly hepatic, but AST can also be found in cardiac and skeletal muscle.
○ Any liver damage from hepatitis, physical trauma (e.g., surgery), ischemia, or injury from some drugs or toxins may elevate AST and ALT.

120
Q
  1. Why are alkaline phosphatase and γGT elevated?
A

○ Slightly different forms of the enzyme alkaline phosphatase (Alk Phos) are present in many tissues including the liver, bile ducts, and bones.
○ The enzyme gamma-glutamyl transferase (γGT) is also present in many tissues including the bile duct, pancreas, gallbladder, and kidneys.
○ Both Alk Phos and γGT are often elevated together in diseases of the biliary tract.
○ Due to γGT’s role in drug detoxification, it can be raised by large amounts
of alcohol ingestion, although it is not specific to alcohol.

121
Q

A 75-year-old man is scheduled to undergo urgent laparotomy for small bowel
obstruction. On preoperative examination he looks jaundiced, and you request a full
liver function test profile including total bilirubin, AST, ALT, Alk Phos, gamma-
glutamyl transferase (γGT), and glucose. The results show:
Why order blood glucose?

A

○ If there is severe liver damage, blood glucose can fall as hepatic gluconeogenesis—the liver’s ability to produce glucose from noncarbohydrates—goes down, although this is a late feature.

122
Q

A 75-year-old man is scheduled to undergo urgent laparotomy for small bowel
obstruction. On preoperative examination he looks jaundiced, and you request a full
liver function test profile including total bilirubin, AST, ALT, Alk Phos, gamma-
glutamyl transferase (γGT), and glucose. The results show:
Are there any other tests that might reflect liver function?

A

○ Prothrombin time (PT) and its derivative the international normalized ratio (INR) are measures of the extrinsic coagulation pathway.
○ Factors I (fibrinogen), II (pro-thrombin), V, VII, and X are made in the liver. ○ When liver function is significantly reduced, lowered hepatic production of these factors prolongs the PT and raises the INR.
○ Albumin is made in the liver. It transports (hormones, fatty acids, drugs, calcium), buffers plasma pH, and maintains oncotic pressure. Liver disease can result in hypoalbuminemia, although it can also be lost via damaged kidneys, the GI tract (enteropathy), skin (burns), and other conditions. As it is a weak acid, hypoalbuminemia can cause a metabolic alkalosis.

123
Q

A 75-year-old man is scheduled to undergo urgent laparotomy for small bowel
obstruction. On preoperative examination he looks jaundiced, and you request a full
liver function test profile including total bilirubin, AST, ALT, Alk Phos, gamma-
glutamyl transferase (γGT), and glucose. The results show:
What might be the next steps to use or further investigate these results?

A

○ After a history and examination, a liver ultrasound or CT abdomen can image the liver, biliary tree, and surrounding structures and identify any hepatic space occupying lesions.
○ Other tests, for example, for viruses, autoantibodies, or a “fibroscan” (a noninvasive test to quantify liver fibrosis) may be requested as needed.
○ Several liver function tests are used as part of risk scoring systems.
○ The Child-Pugh score considers five factors, three of which assess the synthetic function of the liver (total bilirubin level, serum albumin, and INR) along with two more subjective clinical factors (degree of ascites and hepatic encephalopathy).
○ The Model for End-Stage Liver Disease (“MELD”) score uses bilirubin, creatinine, and the INR.
•These scores have been used to predict mortality in patients with
hepatic cirrhosis, in patients with cirrhosis undergoing abdominal surgery or
hepatic procedures and as part of the assessment for liver transplantation.
○ The King’s College Criteria for transfer to a liver center includes some liver
function tests, which differ depending on the cause of the liver disease.

124
Q

A 63-year-old man with hypertension, diabetes, and generalized weakness presents
for resection of small bowel. The patient’s medications include furosemide, meto-
prolol, and acetazolamide.
Lab values: HR 90, BP 105/65, Sats 96%, Hb 11 g/dL, Hct 31%, Na 130 mEq/L,
K 2.3 mEq/L, and Cr 2.0
His EKG shows the following rhythm
Questions
1. What are your concerns regarding his EKG?

A

○ The clinical presentation along with EKG (Fig. 27.1) features (prominent U
waves and apparent QT/U prolongation) suggests significant hypokalemia.
○ Hypokalemia is defined as plasma potassium of less than 3.5 mEq/L.
○ For every 0.3 mEq/L decrease in plasma potassium, the total body potassium stores decrease by 100 mEq/L.
•Mild hypokalemia is serum potassium >2.0 mEq/L;
• Severe hypokalemia is serum potassium <2.0 mEq/L.
The electrocardiographic changes include:
• Early—decrease in T wave.
• Later—ST depression and T inversion.
• PR interval prolongation.
• U waves appear in mid precordial leads.
• Severe—U and T fuse-producing giant U waves and apparent prolongation
of QT interval which is actually QU interval.

125
Q

A 63-year-old man with hypertension, diabetes, and generalized weakness presents
for resection of small bowel. The patient’s medications include furosemide, meto-
prolol, and acetazolamide.
Lab values: HR 90, BP 105/65, Sats 96%, Hb 11 g/dL, Hct 31%, Na 130 mEq/L,
K 2.3 mEq/L, and Cr 2.0
His EKG shows the following rhythm
2. What are the causes for this abnormality?

A
126
Q

How do you emergently correct low potassium preoperatively in this patient?

A

○ Hypokalemia treatment consists of oral or intravenous replacement of
potassium.
• Mild hypokalemia (>2.0 mEq/L): infuse potassium chloride up to 10 mEq/h iv
• Severe hypokalemia (<2.0 mEq/L, ECG changes, intense skeletal muscle weakness): infuse potassium chloride up to 40 mEq/h iv, with continuous ECG monitor.
• Total KCL required is determined by calculating the K deficit.
K deficit (mEq/L) = (Goal K – Measured K)/serum Creatinine × 100
K deficit (mEq/L) = (4.0−2.3)/2 × 100
= 1.7/2 × 100 = 85 mEq/L
= 85 mEq/L

127
Q
  1. How do you monitor and manage perioperatively? Hypokalemic
A

(a) Monitoring
• EKG
• Plasma potassium levels
• ABG
• Peripheral nerve stimulator
(b) Maintenance
• Avoid hyperventilation.
• Avoid hyperglycemia.
• Avoid epinephrine and other beta-2 agonist.
• Avoid diuretics unless supplemented with potassium chloride

128
Q

What are the anticipated problems and concerns anesthetizing this patient? Hypokalaemia

A

(a) Severe hypokalemia may lead to arrhythmias, ventricular tachycardia, and ventricular fibrillation.
(b) Hypokalemic patients may be sensitive to vasodilators or cardiac-depressant effects of volatile anesthetics.
(c) Potential for prolonged response to non-depolarizing muscle relaxants.
(d) Digoxin toxicity can occur with low potassium levels.
(e) Insulin therapy can lower potassium levels.
(f) While treating hypokalemia, concurrent hypomagnesemia should also be
corrected.